PDA

نسخه کامل مشاهده نسخه کامل : اتاق ریاضیات(طرح سؤالات)



صفحه ها : 1 2 3 4 5 [6] 7 8 9 10 11 12 13 14 15 16 17 18 19 20

Boye_Gan2m
22-03-2008, 21:42
میشه جواب این سوال رو بدین و کلا این تیپ سوالات رو توضیح بدین؟!

یک کتاب n صفحه دارد و در شماره گذاری صفحه های آن 3n رقم به کار رفته است.n کدام است؟

Blossom
23-03-2008, 14:47
سلام!
آره با شما درباره matlab موافقم (البته كارهاي اينچنيني رو بكمك kernel برنامه Maple انجام ميده). اينكه فرموديد( ln(1-x ، فكر نكنم چون همون اوليه جواب ميده!

ممنون

یه سوال که به این تاپیک ربطی نداره!
من متلب رو گرفتم ولی بلد نیستم باش انتگرال حساب کنم. می شه راهنماییم کنید؟

mohammad96
23-03-2008, 19:56
یه سوال که به این تاپیک ربطی نداره!
من متلب رو گرفتم ولی بلد نیستم باش انتگرال حساب کنم. می شه راهنماییم کنید؟

سلام! البته که میشه :20:
یکی از toolbox های برنامه MATLAB ، تول باکس symbolic math toolbox هستش که البته قسمت help اون توضیحات کافی رو داره. به طور خلاصه : انتگرال گیری با تابع int انجام میشه. برای مثال :

برای مشاهده محتوا ، لطفا وارد شوید یا ثبت نام کنید

که میشه :

برای مشاهده محتوا ، لطفا وارد شوید یا ثبت نام کنید

دستور syms هم برای اینه که به matlab بفهمونه پارامترهای بعد از اون از نوع symbolic هستن. بد نیست اضافه کنم که دستورهایی مثل simple, pretty و ... هم هستند برای ساده کردن و نمایش زیبا تر نتایج.

با سپاس!

saeed666
23-03-2008, 20:00
سلام!
نه matlab تونست حلش کنه نه maple ؟؟!!! maple میگه از نوع Abel-second kind-class A ولی ... حلش نمیکنه.

ممنون محمد جان ...

Blossom
24-03-2008, 13:30
سلام! البته که میشه :20:
یکی از toolbox های برنامه MATLAB ، تول باکس symbolic math toolbox هستش که البته قسمت help اون توضیحات کافی رو داره. به طور خلاصه : انتگرال گیری با تابع int انجام میشه. برای مثال :

برای مشاهده محتوا ، لطفا وارد شوید یا ثبت نام کنید

که میشه :

برای مشاهده محتوا ، لطفا وارد شوید یا ثبت نام کنید

دستور syms هم برای اینه که به matlab بفهمونه پارامترهای بعد از اون از نوع symbolic هستن. بد نیست اضافه کنم که دستورهایی مثل simple, pretty و ... هم هستند برای ساده کردن و نمایش زیبا تر نتایج.

با سپاس!

مرسی، جالبه.
یه خواهش دیگه، می شه در مورد رسم نتیجه هم توضیح بدین؟
فکر کنم دستور plot هستش.

mohammad96
24-03-2008, 15:41
مرسی، جالبه.
یه خواهش دیگه، می شه در مورد رسم نتیجه هم توضیح بدین؟
فکر کنم دستور plot هستش.

سلام!
اگه بخواييم نمودار توابع رو رسم كنيم، از توابعي مانند

ezcontour
ezcontourf
ezmesh
ezmeshc
ezplot
ezplot3
ezpolar
ezsurf
ezsurfc
fplot

استفاده ميشه. مثلا دستور plot كه فرموديد، براي رسم نمودار هاي دو بعدي استفاده ميشه وقتي كه براي هر كدوم از پارامترها "داده" داشته باشيم، مثلا :


برای مشاهده محتوا ، لطفا وارد شوید یا ثبت نام کنید

از اين نوع توابع مثلا plot3,polar,contour,mesh,surf,isosurface و ... را ميشه نام برد.

Blossom
25-03-2008, 13:42
به این تابع نگاه کنید:

برای مشاهده محتوا ، لطفا وارد شوید یا ثبت نام کنید
می خوام بر حسب t بشه و نه v. چیکار کنم؟
m,g,k هم ثابت هستن.

mohammad96
26-03-2008, 21:22
به این تابع نگاه کنید:

برای مشاهده محتوا ، لطفا وارد شوید یا ثبت نام کنید
می خوام بر حسب t بشه و نه v. چیکار کنم؟
m,g,k هم ثابت هستن.

سلام!
من نتونستم ( با استفاده از Matlab & Maple) يك معادله سرراست بر حسب v بدست بيارم . فكر كنم براي يافتن مقادير v به ازاي t ، بايد به v مقدار داد و كم كم t مورد نظر رو بدست آورد.

ممنون!

Blossom
28-03-2008, 16:16
سلام!
من نتونستم ( با استفاده از Matlab & Maple) يك معادله سرراست بر حسب v بدست بيارم . فكر كنم براي يافتن مقادير v به ازاي t ، بايد به v مقدار داد و كم كم t مورد نظر رو بدست آورد.

ممنون!

خب من باید از v نسبت به t انتگرال و مشتق بگیرم! با مقدار دهی که نمی شه!

mohammad96
28-03-2008, 20:18
خب من باید از v نسبت به t انتگرال و مشتق بگیرم! با مقدار دهی که نمی شه!

با سلام!
چون طرف راست تساوي برابر t است و v هم خود تابع t ، پس عبارت طرف راست تساوي تايع معكوس v مي باشد. حالا براي محاسبه مشتق داريم:

[ برای مشاهده لینک ، لطفا با نام کاربری خود وارد شوید یا ثبت نام کنید ]

براي انتگرال هم فرمولي كه بدست آوردم اينه فقط تابع بايد در بازه انتگرال گيري يك به يك باشه :

[ برای مشاهده لینک ، لطفا با نام کاربری خود وارد شوید یا ثبت نام کنید ]

كه مقادير (v(x بايد يه جوري بدست بياد !

Blossom
29-03-2008, 03:18
من که نفهمیدم!

mohammad96
29-03-2008, 12:13
من که نفهمیدم!

كجاش رو متوجه نشديد؟
رابطه مربوط به مشتق از قاعده زنجيره اي بدست مياد.
ميشه در باره فرمول توضيح بديد كه از كجا بدست اومده و اينكه مقادير k,g,m چقدر هستند؟

ممنون!

matin_blue
30-03-2008, 18:39
سلام
چند تا سوال از حد داشتم اگه کسی جواب بده ممنون می شم
1.عدد یا صفر حدی به روی صفر مطلق مبهمه؟ یا بی نهایت میشه؟
2.عدد به توان مثبت بی نهایت چی میشه؟ عدد به توان منفی بی نهایت چطور؟
3.یک به روی منفی بی نهایت چی میشه؟
4.منفی بی نهایت و مثبت بی نهایت به توان عدد؟؟
5.فقط توابعی که کسری هستند می توانند مجانب قائم داشته باشند؟ اگر نه چه جوری میشه مجانبش رو پیدا کرد؟مثلا: 0=y3 x2 +y2 x +6x3 –y3 +2x-1
6.برای به دست آوردن مجانب قائم میشه اول کسر رو ساده کرد و بعد مجانب کسر ساده شده رو به دست آورد؟
همین!!
بازم ممنون. فعلا...

Majid ²
30-03-2008, 18:48
1.عدد یا صفر حدی به روی صفر مطلق مبهمه؟ یا بی نهایت میشه؟عدد به روی صفر بینهایت میشه


2.عدد به توان مثبت بی نهایت چی میشه؟ عدد به توان منفی بی نهایت چطور؟میشه بینهایت ( حالا اگه مثبت - مثبت و اگه منفی - صفر )


3.یک به روی منفی بی نهایت چی میشه؟عدد روی بینهایت میشه صفر

4 رو بلد نیستم

5 و 6 به کل نمیدونم مجانب قائم چیه
امیدوارم کمکی کرده باشم

matin_blue
30-03-2008, 19:57
ممنون
در مورد سوال اول: عدد به روی صفر حدی با عدد به روی صفر مطلق ینی فرقی نداره؟؟

alex_pk
30-03-2008, 20:31
عدد به توان منفی بی نهایت چطور؟


میشه بینهایت

فكر ميكنم اين ميشه صفر... نه بينهايت.

pp8khat
30-03-2008, 21:36
سلام.
هر گونه سوال مستقیمی باید در تاپیک "طرح سوالات"مطرح شود.لطفاً رعایت کنید.


فكر ميكنم اين ميشه صفر... نه بينهايت.
فکر کنم درست فکر کردید!



1.عدد یا صفر حدی به روی صفر مطلق مبهمه؟ یا بی نهایت میشه؟

من مباحث حد رو خوب بلد نیستم ولی فکر کنم هر چیزی بر روی صفر مطلق می شه مبهم.



2.عدد به توان مثبت بی نهایت چی میشه؟ عدد به توان منفی بی نهایت چطور؟

عدد به توان مثبت بی نهایت فکر کنم می شه بی نهایت.
عدد به توان منفی می نهایت فکر کنم می شه صفر.
آقای PBLover:
علامت توان که ربطی به علامت پایه ندارد.
مثلاً دو به توان منفی 1 می شه 1/2-؟؟ نه..می شه 1/2+
حالا اگه فهمیدید بگید( 2- )به توان 2 می شه 1/4 یا 1/4- یا 4 یا 4-؟؟
بیشتر دقت کنید.



3.یک به روی منفی بی نهایت چی میشه؟

یک بر روی منفی بی نهایت می شه صفر[البته فکر کنم]



4.منفی بی نهایت و مثبت بی نهایت به توان عدد؟؟

اول از فکر کنم فاکتور می گیرم!!:
فکر کنم(
مثبت بی نهایت به توان عدد مثبت می شه مثبت بی نهایت
مثبت بی نهایت به توان عدد منفی می شه صفر
منفی بی نهایت به توان عدد مثبت و زوج می شه مثبت بی نهایت
منفی بی نهایت به توان عدد مثبت و فرد می شه منفی بی نهایت
منفی بی نهایت به توان عدد منفی می شه صفر
)


.فقط توابعی که کسری هستند می توانند مجانب قائم داشته باشند؟ اگر نه چه جوری میشه مجانبش رو پیدا کرد؟

نه خیر...توابع نمایی(لگاریتمی) هم مجانب قائم دارند.برای اطلاعات بیشتر به کتاب ریاضی عمومی پیش دانشگاهی رشته تجربی مراجعه کنید.اگر هم نمی خواید،کتاب ریاضی سال دوم یه چیزایی دربارش نوشته ولی بحث مجانب ها رو باز نکرده.
در ضمن به هیچ یک از پاسخ های من اعتماد نکنید.من مبحث حد رو خودم خونده بودم و فقط یه چیزایی دستگیرم شد.
موفق باشید.

soheilsmart
30-03-2008, 22:01
سلام
چند تا سوال از حد داشتم اگه کسی جواب بده ممنون می شم
1.عدد یا صفر حدی به روی صفر مطلق مبهمه؟ یا بی نهایت میشه؟
2.عدد به توان مثبت بی نهایت چی میشه؟ عدد به توان منفی بی نهایت چطور؟
3.یک به روی منفی بی نهایت چی میشه؟
4.منفی بی نهایت و مثبت بی نهایت به توان عدد؟؟
5.فقط توابعی که کسری هستند می توانند مجانب قائم داشته باشند؟ اگر نه چه جوری میشه مجانبش رو پیدا کرد؟مثلا: 0=y3 x2 +y2 x +6x3 –y3 +2x-1
6.برای به دست آوردن مجانب قائم میشه اول کسر رو ساده کرد و بعد مجانب کسر ساده شده رو به دست آورد؟
همین!!
بازم ممنون. فعلا...


به ترتیب جواب میدم

1-تعریف نشده
2- بستگی داره به عددتون


3.صفر نسبی

4.اینم بستگی به عددتون داره

5. log مجانب قائم داره
سوال 5 رو واضح تر بیان کنید اونها ضرایب هستند یا توان!!!؟؟
6.منظورتون اینیکه مثلا یه عامل صفر شونده داشته باشیم که مثلا در مخرج توانش 3 باشه و در صورت 4 اگه اینطوریه میشه
فقط باید توجه کنید که بعضی وقتا هر چیز مخرجو صفر میکنه مجانب قائم نیست

soheilsmart
30-03-2008, 22:05
فكر ميكنم اين ميشه صفر... نه بينهايت.
بستگی داره
0.5 به توان منفی بی نهایت میشه بی نهایت

در حاکی که 2 به توان منفی بی نهایت میشه صفر

soheilsmart
30-03-2008, 22:06
ممنون
در مورد سوال اول: عدد به روی صفر حدی با عدد به روی صفر مطلق ینی فرقی نداره؟؟
صد در صد فرق داره
عدد به صفر نسبی میشه بی نهایت
در حاکی که عدد به صفر مطلق تعریف نشدست

mass0ood
31-03-2008, 08:49
برای به دست آوردن مجانب ها در توابع کسری ریشه های مخرج معمولا می تونن کاندید مجانب قائم باشن
در توابع لگاریتمی به صورت log v ریشه های v=0 و اگر به صورت log u/v باشه ریشه های u=0 و v=0 کاندید مجانب قائم هستن...(u و v تابع هستن)
خدمت شما دوست عزیز هم عرض کنم که توابع چند جمله ای(کثیر الجمله ای) مجانب قائم ندارن
همیشه یادتون باشه عدد بر روی صفرحدی می شه بینهایت و عدد بر روی بینهایت می شه صفر

Majid ²
31-03-2008, 16:30
آقای PBLover:
علامت توان که ربطی به علامت پایه ندارد.
مثلاً دو به توان منفی 1 می شه 1/2-؟؟ نه..می شه 1/2+
حالا اگه فهمیدید بگید( 2- )به توان 2 می شه 1/4 یا 1/4- یا 4 یا 4-؟؟
بیشتر دقت کنید.



والا من اینجا ابنجوری یاد گرفتم
که عدد به توان بینهایت میشه بینهایت
البته در مورد مثبت بینهایت

Sub007
01-04-2008, 00:15
سلام
چند تا سوال از حد داشتم اگه کسی جواب بده ممنون می شم
1.عدد یا صفر حدی به روی صفر مطلق مبهمه؟ یا بی نهایت میشه؟
2.عدد به توان مثبت بی نهایت چی میشه؟ عدد به توان منفی بی نهایت چطور؟
3.یک به روی منفی بی نهایت چی میشه؟
4.منفی بی نهایت و مثبت بی نهایت به توان عدد؟؟
5.فقط توابعی که کسری هستند می توانند مجانب قائم داشته باشند؟ اگر نه چه جوری میشه مجانبش رو پیدا کرد؟مثلا: 0=y3 x2 +y2 x +6x3 –y3 +2x-1
6.برای به دست آوردن مجانب قائم میشه اول کسر رو ساده کرد و بعد مجانب کسر ساده شده رو به دست آورد؟
همین!!
بازم ممنون. فعلا...

۱) هر گاه صفر مطلق در مخرج قرار بگیرد یعنی تابع در اطراف نقطه‌ای که شما دارید حد میگیرین وجود نداره.

۲) در این مورد اگه منظورت توابع نمایی باشه ۴ حالت پیدا میکنیم:۱)عدد به توان مثبت بینهایت(با شرط اینکه عدد بزرگتر از ۱ باشد) که در این حالت جواب مثبت بینهایت میشه.۲)عدد به توان مثبت بینهایت(با شرط اینکه عدد بین صفر و ۱ باشد) که در این حالت جواب صفر میشه.۳)عدد به توان منفی بینهایت(با شرط اینکه عدد بزرگتر از ۱ باشد) که در این حالت جواب صفر میشه.۴)عدد به توان منفی بینهایت(با شرط اینکه عدد بین صفر و ۱ باشد) که در این حالت جواب مثبت بینهایت میشه.

۳) جواب میشه صفر

۴) اینو نمیدونم.

۵) در توابع کسری معمولاً ریشه مخرج مجانب قائم است.در توابعی که درجه صورت یک واحد از درجه مخرج کوچکتر است امکان وجود مجانب مایل وجود دارد.برای مجانب افقی هم باید تابع رو به سمت مثبت و منفی بینهایت میل بدی.توابع log هم دارای مجانب قائم هستند و توابع نمایی هم معممولاً دارای مجانب افقی هستند.

۶)چون حالتی از حد گیری است این کار ممکن میباشد.

mass0ood
02-04-2008, 23:45
با تشکر از Sub عزیز، من به توضیحاتشون اضافه کنم که در توابع لگاریتمی به صورت log v ، ریشه های v=0 و در توابع به صورت log v/u ریشه های u=0 و v=0 می تونن کاندید مجانب قائم باشن.
در توابع کسری معمولا ریشه های مخرج می تونن کاندید مجانب قائم باشن(که بیشتر در توابع هموگرافیک دیده می شه)
البته قبلا هم فکر کنم گفته بودم!!!!!!!!)

amin_metal1370
03-04-2008, 00:09
1.عدد یا صفر حدی به روی صفر مطلق مبهمه؟ یا بی نهایت میشه؟
عد بر صفر مطلق تعریف نشده است.

3.یک به روی منفی بی نهایت چی میشه؟
عدد بر بی نهایت برابر صفر است

5.فقط توابعی که کسری هستند می توانند مجانب قائم داشته باشند؟ اگر نه چه جوری میشه مجانبش رو پیدا کرد؟مثلا: 0=y3 x2 +y2 x +6x3 –y3 +2x-1
بعضی از توابع غیر گویا نوعی مجانب به نام مجانب مایل دارند.

khatarat
03-04-2008, 09:20
سلام
[ برای مشاهده لینک ، لطفا با نام کاربری خود وارد شوید یا ثبت نام کنید ]


با توجه به دلتا،ميتوان جوابهاي T را بدست آوردو پس از آن با قرار دادن در فرمولهاي 2و3و4 ،x,y,z يعني مختصات نقطه خواسته شده بدست مي آيد،مثلا" اگر 'T=T باشدداريم:


[ برای مشاهده لینک ، لطفا با نام کاربری خود وارد شوید یا ثبت نام کنید ]


موفق باشيد.



با سلام
می خواستم بدونم چطور می شه یه دستگاه سه معادله سه مجهول (مثل بالا) رو با نرم افزار matlab محاسبه کرد؟
خیلی فوری لازم دارم با تشکر

sanih
04-04-2008, 20:35
به نام خدا

سلام بعد از نمی دونم چند ماه دوباره هوس نوشتن به سرم زد:

یه سئوال جالب دارم کسی میتونه حل کنه؟ حتما میتونید اگه هم نه....نه بابا میتونید.

سئوال از این قراره که هشت جعبه داریم که تو هردومشون شش تا توپ هست.میخواهیم تمام این توپ هارو طوری رنگ بزنیم که هر دو توپ در یک جعبه رنگ های متفاوتی داشته باشند و همچنین هر جفت رنگ حداکثر در یک جعبه ظاهر شده باشند.حد اقل رنگهایی که میتونیم استفاده کنیم چقدر است؟



بعدا" خودم جواب رو میذارم

sanih
04-04-2008, 20:41
حل معادلات چند مجهولی در متلب:

syms x y z
equ=('x+y=5','y+z=4','y=4')
[x y z]=solve(equ)

اینو تو نوت پد کپی کن
اگه خواستی توضیح هم بدم؟

sanih
05-04-2008, 21:46
اگه دوست داشته باشید یه تاپیک بزنید تا کلا" حل معادلات مختلف ریاضی رو تو متلب توضیح بدم.
یه سئوال بذارید یکم فکر کنیم ظاهرا" مخ من تعطیل شده. البته کمی تا مدتی...
چی گفتم چه ربطی به این تاپیک داشت؟

mohsen mafia
06-04-2008, 15:37
تابع f در صفر پیوسته است و همچنین (f ( a+b ) = f (a) * f(b
ثابت کنید تابع f در همه ی نقاط پیوسته است؟
:13::13::13::13::13::13::13::13::13::13::13::13::1 3:

matin_blue
06-04-2008, 16:12
حد این تابع رو کی میدونه؟ براکته یک ایکسم؟؟ ایکس میل می کنه به صفر...

sanih
06-04-2008, 22:01
چپ و راستش مهمه ولی میشه +-بینهایت

sanih
06-04-2008, 22:03
اینم شکلش ........

[ برای مشاهده لینک ، لطفا با نام کاربری خود وارد شوید یا ثبت نام کنید ] ([ برای مشاهده لینک ، لطفا با نام کاربری خود وارد شوید یا ثبت نام کنید ])

sanih
06-04-2008, 22:11
البته این تایع خود براکت نیست ولی حدش تونقاط بین عدد صحیح و عدد صحیح+0.5 دقیقا همون براکته و صفر هم همین طوره !
خیالت راحت درسته!!

Mostafa_Amol
07-04-2008, 11:11
سلام
می خواستم بدونم مشتق این عبارت چی می شه: (X^X^2)

من می گم جوابش این می شه :

(X^X^2)'= ?

طبق : X^U=> U'.(X^-1) :
(X^2)(X^((X^2)-1)
طبق:
(A^x)' = (A^x)Ln(A).x'
آخرش :
 (X^X^2)'= (X^2)(X^((X^2)-1)) (A^x)Ln(A).x'



اما کتاب می گه :

(X^X^2)'= (X^X^2)((X^2)LnX)'=(X^X^2)(2XLn+(X^2).1/X)= (X^X^2)(2XLnX+X)

ولی من دلیلش رو نمی فهمم ....
لطفا اگه من اشتباه می کنم و حق با کتابه یه نفر بهم توضیح بده .
خیلی ممنونم :11:

sherlockholmz
08-04-2008, 08:06
سلام
در مشتق گيري از توابع باتوان متغير لازم نيست فرمول حفظ كني ، از دو طرف لگاريتم بگير و مشتق گيري ساده خودت را انجام بده:



[ برای مشاهده لینک ، لطفا با نام کاربری خود وارد شوید یا ثبت نام کنید ]



موفق باشي

sanih
13-04-2008, 01:12
دوستان یکی یه سئوال رمز گشایی بذاره یکم با ور بریم بابا خسته شدیم از این مشتق و انتگرال....

musa
14-04-2008, 21:21
با سلام

اين سوال در مورد مشتق جزئي هست بي زحمت حل Zy رو ميخوام اگر مقدور هست توضيحش رو هم بديد

Z = ye ^x/y

با تشكر.

pp8khat
14-04-2008, 21:46
دوستان یکی یه سئوال رمز گشایی بذاره یکم با ور بریم بابا خسته شدیم از این مشتق و انتگرال....


با سلام

اين سوال در مورد مشتق جزئي هست بي زحمت حل Zy رو ميخوام اگر مقدور هست توضيحش رو هم بديد

Z = ye ^x/y

با تشكر.

:31::31::18::18::18::27::27::27:
sanih جان! تا هنگ نکردی این مسئله رو حل کن!
چند عدد 4 رقمی وجود دارد به طوری که مجموع ارقامشان بر 6 بخش پذیر باشد؟

sherlockholmz
15-04-2008, 07:39
با سلام

اين سوال در مورد مشتق جزئي هست بي زحمت حل Zy رو ميخوام اگر مقدور هست توضيحش رو هم بديد

Z = ye ^x/y

با تشكر.


باسلام،
مشتق گيري جزئي تفاوت چنداني با مشتق گيري معمولي ندارد،شما فقط بايد در هنگام مشتق گيري تمام متغيرهاي مستقل را به غير از آن متغير كه برحسب آن مشتق گيري انجام مي پذيردثابت فرض كنيد،همين!
مثلا" در اينجا چون مشتق برحسب y گرفته ميشود پس متغير مستقل x بعنوان ثابت در فرمول فرض مي گردد.بقيه مراحل مانند مشتق ساده انجام مي پذيرد:



[ برای مشاهده لینک ، لطفا با نام کاربری خود وارد شوید یا ثبت نام کنید ]



موفق باشيد.

morteza_bayat
15-04-2008, 11:14
1=مبهم
2=عدد به توان مثبت بی نهایت ولی عدد به توان منفی بی نهایت 0 میشه
3=

4=منفی بینهایت به توان عدد میشه صفر
5

soheilsmart
15-04-2008, 16:09
:31::31::18::18::18::27::27::27:
sanih جان! تا هنگ نکردی این مسئله رو حل کن!
چند عدد 4 رقمی وجود دارد به طوری که مجموع ارقامشان بر 6 بخش پذیر باشد؟
یه راهش اینه که عدد رو به صورت
abcd
(یه خط بالاش از a تاd بذارید)
بعد بیام معادله های زیر رو حل کنیم
a+b+c+d=6
یا
a+b+c+D=12
یا
a+b+c+d=18
یا
a+b+c+d=24
یا
a+b+c+d=30
یا
a+b+c+d=36

را با شرط
a که عضو مجموعه اعداد صحیح و بین 1 تا 9 هستش (خود 1و 9 هم شامل میشن اگه 0 بشه عدد 3 رقمی میشه پس غیر قابل قبوله !!!)

و b , c, d عضو اعداد صحیح و بین 0 تا تا 9 هستش (خود صفر و 9 هم هستن)


در حلش از مرحله 2 به بعد باید از شمول و عدم شمول استفاده کنیم

همچنین از تعداد انتخاب k گل از n گل
که مربوط به ریاضیات گسسته میشه
جواب های هر مرحله رو پیدا میکنیم و سرانجام همه را با هم جمع میکنیم

البته این راه طولانیه ولی خوب این راهیه که به ذهنم میرسه

musa
15-04-2008, 20:17
باسلام،
مشتق گيري جزئي تفاوت چنداني با مشتق گيري معمولي ندارد،شما فقط بايد در هنگام مشتق گيري تمام متغيرهاي مستقل را به غير از آن متغير كه برحسب آن مشتق گيري انجام مي پذيردثابت فرض كنيد،همين!
مثلا" در اينجا چون مشتق برحسب y گرفته ميشود پس متغير مستقل x بعنوان ثابت در فرمول فرض مي گردد.بقيه مراحل مانند مشتق ساده انجام مي پذيرد:



[ برای مشاهده لینک ، لطفا با نام کاربری خود وارد شوید یا ثبت نام کنید ]



موفق باشيد.





خيلي ممنون

مشكل اينه كه متوجه نميشم علامت جمع از كجا مياد ؟

sanih
15-04-2008, 22:34
pp8khat جان اون مسئله با همنهشتی حل میشه حلش هم سادس ولی موقتا" تا 2 روز و بعد از او تا 4 روز نمیتونم بیام. ولی جمعه حلش رو واست با توضیح میذارم من 31ام دارم میرم بابل برای مسابقات روبوتیک لیگ مین یاب در ضمن من سرپرست 2 تیم دیگه هم هستم نزدیک های مسابقات باید پیش بچه ها باشی و استرس رو از تیم دور کنی تا اونا با یه آرامش کار کنن.از همه میخوام برامون دعا کنن.

واسه چی اینو نوشتم؟خودمم نفهمیدم

sanih
15-04-2008, 22:37
error.......

sanih
15-04-2008, 22:38
راستی دوستان شما چرا برای حل مشتق ار متلب کمک نمی گیرید:31: اگه طالب هستید من هم پایم تا روش انواع مشتق گیری رو بگم ها:40::40:!

sanih
15-04-2008, 23:08
سهیل سمارت جان ما باید اصل شمول و عدم شمول رو تو 4 مجموعه بررسی کنیم اون هم چندا... که واقعا وحشت ناک میشه و حالا واحشتناکش مونده اگه سئوال میگفت چند عدد 7 رقمی.... چی اون وقت شمول و عدم شمول تو 7 مجموعه که بله دیگه این کار حضرت فیله .من روش حل رو با همنهشتی میگم اون وقت خودت مقایسه کن.

یاحق

sherlockholmz
16-04-2008, 07:50
خيلي ممنون

مشكل اينه كه متوجه نميشم علامت جمع از كجا مياد ؟



باسلام،
علامت جمع از اينجا مي آد كه
"مشتق حاصل ضرب دو جمله برابر است با مشتق جمله اول در جمله دوم بعلاوه مشتق جمله دوم در جمله اول"
اينجا نيز مشتق حاصلضرب دو جمله را مي خواهيم (يكي y و ديگري جمله نمائي) .
اميدوارم مشكلت حل شده باشد.
موفق باشي

Mostafa_Amol
16-04-2008, 15:48
سلام و خیلی متشکرم به خاطر تایپیک خیلی خوبتون و از همه ی کسایی که جواب مارو می دهند

سوالم ( منم توی Ln موندم .... :24:) :

می شه بهم بگید مشتق این دوتا چطور به دست می آد :


y=x^(x^x)


y=a^(x^x)

سپاسگذارم :11:

Blossom
16-04-2008, 17:22
آقا یه سوال! نشان دهید که انتگرال

برای مشاهده محتوا ، لطفا وارد شوید یا ثبت نام کنید
بین ۱ و ۱.۲۵ قرار دارد.

musa
16-04-2008, 20:52
باسلام،
علامت جمع از اينجا مي آد كه
"مشتق حاصل ضرب دو جمله برابر است با مشتق جمله اول در جمله دوم بعلاوه مشتق جمله دوم در جمله اول"
اينجا نيز مشتق حاصلضرب دو جمله را مي خواهيم (يكي y و ديگري جمله نمائي) .
اميدوارم مشكلت حل شده باشد.
موفق باشي

دست شما درد نكنه از اينكه وقت گزاشتيد :11:

sherlockholmz
19-04-2008, 09:08
سلام و خیلی متشکرم به خاطر تایپیک خیلی خوبتون و از همه ی کسایی که جواب مارو می دهند

سوالم ( منم توی Ln موندم .... :24:) :

می شه بهم بگید مشتق این دوتا چطور به دست می آد :


y=x^(x^x)


y=a^(x^x)

سپاسگذارم :11:

سلام
همانطور كه گفتم شما براي سادگي و جلوگيري از اشتباه بهتر است از دو طرف ln بگيري تا رابطه ساده شود و سپس مشتق بگيري مثلا" براي اولي:

[ برای مشاهده لینک ، لطفا با نام کاربری خود وارد شوید یا ثبت نام کنید ]

مي بيني كه براحتي قابل حل است.حال براي امتحان و مرور دومي را خودت حلش كن(البته اگه حوصله نداشتي در همين حل بالائي بجاي x پايه ،ثابت a را بذار و ساده كن.پاسخ بايد چيزي شبيه اين باشد:


[ برای مشاهده لینک ، لطفا با نام کاربری خود وارد شوید یا ثبت نام کنید ]



موفق باشي

Mostafa_Amol
19-04-2008, 12:50
سلام داداش ... خیلی از تون سپاسگذارم

اما اون سوال:
X^(X^X)1
رو کتاب این بدست آورد :

y'=y[(X^X) LnX +(X^X) Ln(^2)X +((X^X)/X)]1
( در یک ضرب کردم تا پرانتزها سر جای خودشون قرار بگیرند)
خوب راستش من نمی تونم متوجه بشم این دوتا جواب شبیه هم هستند؟ ( منظورم اینه که اگه یکدومشونو ساده کنیم ، اون یکی دیگه بدست می آد؟)


جواب دومی هم این طوری بدست آوردم درسته؟
y=a^(X^X)1
Lny=(X^X) Lna
Ln(Lny) = XLnX + Ln(Lna)1
(y'/y)/Lny = LnX +1 + (Lna'/Lna)
y'=yLny(LnX + 1)1
a^(X^X) Lna^(X^X)) (LnX+1)1)

سپاسگذارم

Blossom
19-04-2008, 23:57
آقا یه سوال! نشان دهید که انتگرال

برای مشاهده محتوا ، لطفا وارد شوید یا ثبت نام کنید
بین ۱ و ۱.۲۵ قرار دارد.

البته فراموش کردم بگم که انتگرال بین ۰ و یک منظوره!

sherlockholmz
20-04-2008, 09:33
سلام داداش ... خیلی از تون سپاسگذارم

اما اون سوال:
X^(X^X)1
رو کتاب این بدست آورد :

y'=y[(X^X) LnX +(X^X) Ln(^2)X +((X^X)/X)]1
( در یک ضرب کردم تا پرانتزها سر جای خودشون قرار بگیرند)
خوب راستش من نمی تونم متوجه بشم این دوتا جواب شبیه هم هستند؟ ( منظورم اینه که اگه یکدومشونو ساده کنیم ، اون یکی دیگه بدست می آد؟)


جواب دومی هم این طوری بدست آوردم درسته؟
y=a^(X^X)1
Lny=(X^X) Lna
Ln(Lny) = XLnX + Ln(Lna)1
(y'/y)/Lny = LnX +1 + (Lna'/Lna)
y'=yLny(LnX + 1)1
a^(X^X) Lna^(X^X)) (LnX+1)1)

سپاسگذارم

سلام
بله عزيزم اين جواب و جواب كتاب يكي است و بصورت زير مي تواني از اين جواب به جواب كتاب برسي:

[ برای مشاهده لینک ، لطفا با نام کاربری خود وارد شوید یا ثبت نام کنید ]


پس مي بيني كه جوابها يكي است.
حل دومي هم كاملا" صحيح است. نمرت 20!
موفق باشي

orlan2magic_66
21-04-2008, 11:05
سلام
من بايد كنفرانسي در مورد كواريانس بدم توي كلاس تئوري احتمالات دانشگاه
فرمولي ميدونم چيه اما يه تعريفي در حد 5 خط ازش مي خوام
فقط بدجور لازمم خيلي ممنون

na694
21-04-2008, 23:27
يكي از شما ها كه روي خط هستيد سئوال دارم كي ميتونه جواب بده؟

hamid6885
30-04-2008, 10:42
سلام. لطفا در تصویر زیر رند r به رند x رو حساب کنید. با تشکر

[ برای مشاهده لینک ، لطفا با نام کاربری خود وارد شوید یا ثبت نام کنید ]

venus1247
01-05-2008, 09:47
سلاما

اگر ميشه مشتق فاكتوريل ها رو يكي به من بگه؟
براي مثال مشتق !n :41::20:
ممنون:20:

Blossom
01-05-2008, 09:59
سلاما

اگر ميشه مشتق فاكتوريل ها رو يكي به من بگه؟
براي مثال مشتق !n :41::20:
ممنون:20:

n فاکتوریل یعنی:

برای مشاهده محتوا ، لطفا وارد شوید یا ثبت نام کنید
و ۰ فاکتوریل می‌شه ۱
برای مثال ۱ فاکتوریل می‌شه ۱. دو فاکتوریل می‌شه ۲. ۳ فاکتوریل می‌شه ۶. ۴ فاکتوریل می‌شه ۲۴. ۵ فاکتوریل می‌شه ۱۲۰. ۶ فاکتوریل می‌شه ۷۲۰.

pp8khat
01-05-2008, 12:50
سلاما

اگر ميشه مشتق فاكتوريل ها رو يكي به من بگه؟
براي مثال مشتق !n :41::20:
ممنون:20:

سلام.اتفاقاً این موضوع به تازگی برای من هم سوال بوده...
از یکی از معلمامون پرسیدم بلد نبود؟!!
توی یکی از این نرم افزار های ریاضی نوشتم میگه میشه:
[ برای مشاهده لینک ، لطفا با نام کاربری خود وارد شوید یا ثبت نام کنید ]
من که نفهمیدم یعنی چی؟ شما فهمیدی به ما هم بگو!
با تشکر از باز کردن بحث

amintnt
01-05-2008, 13:44
سلام..... سوالی در یکی از تمارین کتاب جبر و احتمال سال سوم آمده که می گوید: از میان 5 زن و 6 مرد، 5 نفر را انتخاب میکنیم، احتمال آنکه حداقل 4 نفرشان مرد باشد را حساب کنید....... حال سوال بنده این است که چرا نمی توان گفت که ترکیب 4 مرد از 6 مرد را در ترکیب 1 از 7 نفر باقیمانده ضرب و جواب را بر ترکیب 5 از 11 تقسیم میکنیم؟

sherlockholmz
03-05-2008, 09:41
سلاما

اگر ميشه مشتق فاكتوريل ها رو يكي به من بگه؟
براي مثال مشتق !n :41::20:
ممنون:20:


سلام، شمااول به من بگو مشتق n چي مي شود تا من مشتق !n را بهت بگويم!(pay attention plz)

venus1247
03-05-2008, 11:42
عليك سلام ،
دوست عزيز !n مثال بود :13:

منظور من همون فاكتوريل ها بود حالا هر چي دلت ميخواد جاي n بزار و مشتق اون رو بگير

ميتوني اين سوال رو از دوست pp8khat هم بپرسي چون فكر ميكنم ايشون منظور من رو از اين سوال فهميدن:10:

sanih
03-05-2008, 21:54
منظورتون رو خودتون باید بگید نه دوست شما.
nیه عدد یا متغیره که میخوای نسبت به n مشتق بگیری؟

sanih
03-05-2008, 22:00
در ضمن آقای هولمز شرمنده اون معدله رو گذاشتم تو کارهای روزانم ولی وقت نمیشه.واقعا وقت نمیشه. ولی تا چند روز آینده حتما جوابشو براتون میفرستم

یا حق

venus1247
03-05-2008, 22:40
sanih دوست عزيز سلام
n متغير نه عدد (از عدد نميشه مشتق گرفت)
خواهش ميكنم ديگه به اين سوال فكر نكنيد چون من از سوالم پشيمون شدم


دوستان ديگر اگر منظور من رو فهميدن ممنون ميشم جواب من رو بدن اگر هم كسي منظور من رو نفهميده ببخشيد چون بيشتر از اين بلد نيستم توضيح بدم

rouhallah
03-05-2008, 23:49
سلام
من فکر کنم منظورتو فهمیدم
طبق تعریف مشتق میتونم منظورتو بگم
عکس زیر توضیح میده
[ برای مشاهده لینک ، لطفا با نام کاربری خود وارد شوید یا ثبت نام کنید ]

sherlockholmz
04-05-2008, 10:22
عليك سلام ،
دوست عزيز !n مثال بود :13:

منظور من همون فاكتوريل ها بود حالا هر چي دلت ميخواد جاي n بزار و مشتق اون رو بگير

ميتوني اين سوال رو از دوست pp8khat هم بپرسي چون فكر ميكنم ايشون منظور من رو از اين سوال فهميدن:10:


باسلام مجدد،
منظور من يك نكته مهم بود.
همانطور كه مي داني x يك متغير پيوسته است و n يك متغير گسسته،كه در هيچ نقطه ائي پيوسته نيست پس حد نداردوبه طريق اولي مشتق پذير نمي باشد.اين نكته ائي بود كه مي خواستم مشخص شود.البته فاكتوريل را براي تمام اعداد با تابع گاما تعريف كرده اند:


[ برای مشاهده لینک ، لطفا با نام کاربری خود وارد شوید یا ثبت نام کنید ]


كه اين فرمول براي اعداد صحيح !n را بصورت زير محاسبه مي كند:



[ برای مشاهده لینک ، لطفا با نام کاربری خود وارد شوید یا ثبت نام کنید ]


حال اين تابع در تمام نقاط تعريف شده و پيوسته است و مشتق پذير اما فاكتوريل معروف !n در هيچ نقطه ائي مشتق پذير نمي باشد.
موفق باشيد.

venus1247
04-05-2008, 12:27
آقاي sherlockholmz عزيز ممنون از اين جوابتون:10:

امروز صبح اين سوال رو از استادم پرسيدم همين جواب رو دادن

بسيار سپاس گذارم:11:

trainercodes
04-05-2008, 15:51
یک مسئله دارم که اگه میشه حل کنید:
ایا تعداد اعداداول به شکل 2به توان k به اضافه یک که k به صورت 2به توان m باشد بی نهایت است؟
---m
-----2
--1+2
یعنی عدد مورد نظر به شکل بالا باشد که در ان m در اعداد طبیعی است.
(مثلا به ازای m=3 عدد مورد نظر 257 میشود)

trainercodes
04-05-2008, 16:05
اقا یکی این مشتق رو برای من توضیح بده!!!!!!!!!!!!!!!!!!!!!!!!!!!!!!!!!!!!!!!!!!!! !!!!!!!!!!!!!!!!!!!!!!!!!!!!!!

mehrdad_ati
04-05-2008, 17:57
سلام به همگی 2 سوال داشتم

من سال دوم دبیرستان هستم و تو یکی از مدارس استعدادهای درخشان درس مسخونم و معلم ریاضیمون از ریاضی تکمیلی 2 چیز زیادی بهمون نگفته و امسال هم ریاضی هماهنگ سمپاد شده میخواستم اگر میشه این دو سوال رو برام حل کنید

بخش بردارها - ریاضیات تکمیلی 2 - صفحه 95

سوال6 - بردارهای a و b باید چه ویژگی داشته باشند تا a+b و a-b بر هم عمود باشند؟

سوال 7- بردارهای a و b باد چه ویژگی داشته باشند تا a+b نیمساز زاویه میان a و b باشد؟

سوال 8 - بردار های a و b باید چه ویژگی داشته باشد تا

الف) a+b =a-b
ب) a+b>a-b
ج) a+b<a-b

خواهشا اینا رو جواب بدید که تمام مشکلات بردارهام تو همینا نهفته است! تا امشب نیازشون دارم با تشکر

sanih
04-05-2008, 21:53
sanih دوست عزيز سلام
n متغير نه عدد (از عدد نميشه مشتق گرفت)
خواهش ميكنم ديگه به اين سوال فكر نكنيد چون من از سوالم پشيمون شدم


دوستان ديگر اگر منظور من رو فهميدن ممنون ميشم جواب من رو بدن اگر هم كسي منظور من رو نفهميده ببخشيد چون بيشتر از اين بلد نيستم توضيح بدم


سلام دوست من،منظور من این نبود که برات از یه عدد مشتق بگیرم منظور من این بود که منظور تو چیه؟(چقدر فلسفی).

و نکته دیگه اینه که از عدد میشه مشتق گرفت و مشتق اون صفره(البته این جواب جسارت به اساتید تاپیکه)با توجه به تعریف مشتق که شیب خط مماس بروی نموداردر یک نقطه اس(واقعا" نقطه اس؟) میتوان گفت که شیب y=cیه عدده که یه برابر باصفره.

یا حق

sanih
04-05-2008, 22:01
برو بچ یه سر به اینجل بزنید و نظراتون رو بگید.

[ برای مشاهده لینک ، لطفا با نام کاربری خود وارد شوید یا ثبت نام کنید ]

sanih
05-05-2008, 19:54
[ برای مشاهده لینک ، لطفا با نام کاربری خود وارد شوید یا ثبت نام کنید ]

venus1247
06-05-2008, 17:03
سلام
از دوستان خواهش ميكنم اگر مطلبي در مورد (ماتريس واندرموند ) ميدونيد به من كمك كنيد
سپاسگذارم

sanih
06-05-2008, 23:06
[ برای مشاهده لینک ، لطفا با نام کاربری خود وارد شوید یا ثبت نام کنید ]

sanih
06-05-2008, 23:13
ماتریس واندر موند:
[ برای مشاهده لینک ، لطفا با نام کاربری خود وارد شوید یا ثبت نام کنید ] ([ برای مشاهده لینک ، لطفا با نام کاربری خود وارد شوید یا ثبت نام کنید ])

hlpmostafa
08-05-2008, 01:51
سلام
سوال من در مورد یک اسکالر هست:
می دونید که اسکالر این طور تعریف می شه:کمیتی که تحت چرخش محورها ناوردا باشد.
همچنین تعریف بردار هم همینه.
حالا از کجا معلوم، حاصل ضرب اسکالر دو بردار، که تحت چرخش محورهای مختصات ناورداست،یک بردار نیست؟

sanih
13-05-2008, 01:11
امروز یه سئوال دارم براتون بچه هایی که میخوان بهمن ماه المپیاد شرکت کنن اینو حتما بخونن. این یکی از مسائلیه که من خیلی دوسش داشتم حالا واستون میذارمش:
ثابت کنید کنید معدله زیر در مجموعه اعداد طبیعی جواب نداره:
4xy-x-y=z^2
شاید فردا حلشو گذاشتم.

یاحق

sanih
13-05-2008, 01:20
ثابت کنید کسر زیر به ازای هر x عضو Nتحویل ناپذیر است:

14x+3
--------
21x+4

sanih
13-05-2008, 01:31
a1, a2, ... , an, b1, ... , bn
اعداد حقیقی غیر منفی هستند،نشان بدهید که :

(∏ ai)1/n + (∏bi)1/n ≤ (∏(ai+bi))1/n

sanih
13-05-2008, 01:48
error..........

zahedy2006
13-05-2008, 11:37
a1, a2, ... , an, b1, ... , bn
اعداد حقیقی غیر منفی هستند،نشان بدهید که :

(∏ ai)1/n + (∏bi)1/n ≤ (∏(ai+bi))1/n


آقا اینو تو Microsoft equation برنامه word تایپ کن بعد تو paint کپی شو ذخیره کن بعد آپلود کن. الان پرانتزا جابه جا شده فهمیدنش سخته.



ثابت کنید کسر زیر به ازای هر x عضو Nتحویل ناپذیر است:

14x+3
--------
21x+4


ب م م بگیریم میشه
ابتدا 2 برابر پایینی را به بالایی اضافه می کنیم (در ب م م گیری) میشه 56x+11 و این یکی هم که 14x+3 است حالا 4 برابر 14x+3 را از این کم می کنیم میشه 1
پس داریم 14x+3,1 که همون یک است پس ساده تر نمیشه

sanih
14-05-2008, 22:28
ب م م بگیریم میشه
ابتدا 2 برابر پایینی را به بالایی اضافه می کنیم (در ب م م گیری) میشه 56x+11 و این یکی هم که 14x+3 است حالا 4 برابر 14x+3 را از این کم می کنیم میشه 1
پس داریم 14x+3,1 که همون یک است پس ساده تر نمیشه
آفرین درسته.

واسه این هم یه پرانتز بزار اولش!


(∏ ai)1/n + (∏bi)1/n ≤ (∏(ai+bi))1/n

یاحق.

zahedy2006
15-05-2008, 10:40
فکر کنم صورت سوال رو اشتباه می کنم چون اینی که من فهمیدم که خیلی آسون حل میشه

[ برای مشاهده لینک ، لطفا با نام کاربری خود وارد شوید یا ثبت نام کنید ]
R چیست؟
R حاصلضربی از اعداد ai و bi هست که چون هیچکدام منفی نیستند پس این حاصلضرب بزرگتر مساوی صفره.

حالا اگه صورت درست را می تونید با ورد تایپ کنید بهتره.

راستی: اسم این قضیه همون چیز نبود. چیز ... چیز ...
یادم اومد: قضیه قدر نسبت هندسی مجموع و جزء

emad62
15-05-2008, 21:50
سلام

من دانشجوی الکترونیک هستم. این ترم درس آمار دارم. و خیلی هم ازش خوشم نمیاد. از شانس توپ ما یکی دوهفته پیش استاد گیر داد به ما که بیا مسئله حل کن! ما هم نرفتیم ! چون چیزی ازون مطلب بلد نبودم.استاد هم لطف فرموده و گفت شما بهتره درس رو حذف کنید چون یکی دوجلسه هم غایب بودی!!!:19: به همین سادگی:18: بعد از کلی کلنجار با استاد قبول کرد به شرط حل چند تا تمرین از حذف ما بگذره. اما هرچی با این سوال ور میرم به نتیجه نمیرسم. گفتم شاید کسی از دوستان بتونه این مسئله رو حل کنه و ما رو نجات بده. مطمئن باشید دعاتون میکنم اگه کمکم کنید:

سوال:
در ظرفی 52 کارت به 4 رنگ مختلف که هر کدام از رنگها از 1 تا 13 شماره گذاری شده اند ، 2 توپ به تصادف و بدون جایگذاری انتخاب میکنیم اگر متغیر تصادفی Z تعداد 1 (تک) هایی باشد که در بار اول استخراج شده باشد و متغیر W تعداد 1(تک)هایی باشد که در هر دو بار مشاهده شده باشد:
الف)تابع احتمال توام Zو W را پیدا کنید

ب)تابعهای کناری [ (F(W) , F(Z ] , را بیابید ونیز توزیع شرطی W/Z=1 (د Wبه شرط Z مساوی 1) را بیابید.

دوستان من فقط یکی دو روز دیگه فرصت دارم/ جداً منو نجات دادید اگه این مسئله رو حل کنید.
درضمن عذر میخوام اگه این مسئله رو اینجا مطرح میکنم

babak_kiyani
18-05-2008, 23:04
دوست عزيز چند تا از سوالات شما نادرست مي باشد. سوالهاي يك و دو و سه فقط در مبحث حد ظاهر مي شوند . و الا چون بي نهايت يك مفهوم است و عدد نيست نمي توان سوالهايي به اين شكل كه بي نهايت به توان عدد چي ميشه را مطرح كرد. اگر منظورتون تو مبحث حد هستش در مورد سوال دو شما عرض كنم كه عدد بتوان مثبت بي نهايت اگر عدد مثبت و بزرگتر از يك باشد حا صل مثبت بي نهايت است . اگر عدد بين صفر و يك باشد حاصل صفر است و اگر عدد يك باشد حاصل مبهم است. براي اعداد منفي توابع نمائي تعريف نشده اند لذا عدد منفي به توان يك عدد تعرف نشده مي باشد.. در مورد سوال سه شما هم عرض كنم كه بطور كلي عدد بر بي نهايت ( چه مثبت و چه منفي ) در مبحث حد برابر صفر است. سوال چها شما هم جزو حالتهاي مبهم در حد مي باشد كه بايد رفع ابهام شود. براي فهم بهتر اين مسائل راهنمايي مي كنم كه از كتاب حساب ديفرانسيل و انتگرال سيلورمن جلد اول استفاده كنيد. موفق باشيد.

babak_kiyani
18-05-2008, 23:06
1=مبهم
2=عدد به توان مثبت بی نهایت ولی عدد به توان منفی بی نهایت 0 میشه
3=

4=منفی بینهایت به توان عدد میشه صفر
5

دوست عزيز منفي بينهايت به توان مثلا 2 مي شه منفي بينهايت ضربدر منفي بي نهايت كه همه مي دانيم مبهم است.پس صفر نمي شود.

Aryan.kh
25-05-2008, 09:50
سلام من می خواستم بدونم چه طوری میشه با نرم افزار maple یا mathlab رویه هایی رو ترسیم کرد این پروژه درس ریاضی 2 مون هست اگر میشه یه کمکی بکنید

Mostafa_Amol
26-05-2008, 11:10
سلام

اگر Sin a = ((a^4)+1)/2a^2 باشد Cos2a=?

آخرای جوابش این می شه که من اصلا دلیلش رو متوجه نمی شم:
a^2= - + 1 => Cos2a= -1

اگه بهم بگید دلیلش چیه خیلی ممنون می شم
سپاسگذارم
یاحق

یکتا ب
26-05-2008, 21:36
1-هر وقت صفر مطلق مخرج باشه مبهمه یعنی تعریف نشده
2- اولی مثبت بی نهاته دومی صفر
3- صفر
4- حالتهای زیادی داره باید روی عدد بحث کرد .
5- نه مثلا تانژانت هم مجانب داره
6-ممکنه کسر مجانب دار رو ساده کرد دیگه مجانب نداشته باشه پس قبل از ساده کردن باید به دست بیاریم.

the dead
28-05-2008, 12:42
اگر z تابعی بر حسب x،y باشد که در معادله f(x,y,z)=0 به هم مربوط شده باشند انگاه
مشتقات ضمنی زير را اثبات کنيد
[ برای مشاهده لینک ، لطفا با نام کاربری خود وارد شوید یا ثبت نام کنید ]

[ برای مشاهده لینک ، لطفا با نام کاربری خود وارد شوید یا ثبت نام کنید ]
مرسی

sherlockholmz
31-05-2008, 09:09
سلام

اگر Sin a = ((a^4)+1)/2a^2 باشد Cos2a=?

آخرای جوابش این می شه که من اصلا دلیلش رو متوجه نمی شم:
a^2= - + 1 => Cos2a= -1

اگه بهم بگید دلیلش چیه خیلی ممنون می شم
سپاسگذارم
یاحق

سلام،
يك نكته جالب توي اين مسئله است:



[ برای مشاهده لینک ، لطفا با نام کاربری خود وارد شوید یا ثبت نام کنید ]




موفق باشيد.

sherlockholmz
31-05-2008, 14:27
اگر z تابعی بر حسب x،y باشد که در معادله f(x,y,z)=0 به هم مربوط شده باشند انگاه

مشتقات ضمنی زير را اثبات کنيد
[ برای مشاهده لینک ، لطفا با نام کاربری خود وارد شوید یا ثبت نام کنید ]

[ برای مشاهده لینک ، لطفا با نام کاربری خود وارد شوید یا ثبت نام کنید ]
مرسی


مستقلند داريم x و y بافرض اينكه متغير م



[ برای مشاهده لینک ، لطفا با نام کاربری خود وارد شوید یا ثبت نام کنید ]

the dead
03-06-2008, 09:03
;;;;;;;;;;;;;;;;;;;;;;;

aidin_mh
04-06-2008, 15:04
عدد روی صفر حدی می شه بینهایت ولی عدد روی صفر مطلق هم وجود ندارد.

lebesgue
06-06-2008, 00:13
سلام
چند تا سوال از حد داشتم اگه کسی جواب بده ممنون می شم
1.عدد یا صفر حدی به روی صفر مطلق مبهمه؟ یا بی نهایت میشه؟
2.عدد به توان مثبت بی نهایت چی میشه؟ عدد به توان منفی بی نهایت چطور؟
3.یک به روی منفی بی نهایت چی میشه؟
4.منفی بی نهایت و مثبت بی نهایت به توان عدد؟؟
5.فقط توابعی که کسری هستند می توانند مجانب قائم داشته باشند؟ اگر نه چه جوری میشه مجانبش رو پیدا کرد؟مثلا: 0=y3 x2 +y2 x +6x3 –y3 +2x-1
6.برای به دست آوردن مجانب قائم میشه اول کسر رو ساده کرد و بعد مجانب کسر ساده شده رو به دست آورد؟
همین!!
بازم ممنون. فعلا...

اول یک نکته رو بگم که، این بینهایتی که در حد مطرح است، با آن بینهایتی که در مورد شمار اعضای مجموعه
های نامتناهی و...(به معنای بیشمار، یا بیپایان)است، تفاوت اساسی دارد.
بطور کلی وقتی گفته میشه که lim x-->x0 f(x) = +binahayat منظور این هست که با نزدیک شدن هر چه
بیشتر x به x0 مقدار تابع یعنی f(x) بطور پیوسته افزایش پیدا میکند.

2.عدد به توان مثبت بی نهایت چی میشه؟ عدد به توان منفی بی نهایت چطور؟
برای هر کدام از این سوالهای بسته به عددتون، چندین حالت پیش میاد که من یکی یکی میگم.
1- عدد بزرگتر از 1 باشد و به توان مثبت بینهایت، جواب: مثبت بینهایت
2- عدد بزرگتر از 1 باشد و به توان منفی بینهایت، جواب: صفر
3- عدد برابر با 1 باشد و به توان مثبت یا منفی بینهایت، جواب: 1
4- عدد بین صفر و یک باشد و به توان مثبت بینهایت، جواب: صفر
5-عدد بین صفر و یک باشد و به توان منفی بینهایت، جواب: مثبت بینهایت
6- عدد بین صفر و منفی 1 باشد و به توان مثبت بینهایت، جواب: صفر
7- عدد بین صفر و منفی 1 باشد و به توان منفی بینهایت، جواب: حد جواب ندارد، زیرا به سمت مقدار مشخصی
میل نمیکند (بسته به زوج یا فرد بودن توان، جواب قرینه میشود(قدر مطلقش به سمت مثبت بینهایت میرود))
8-عدد برابر با منفی 1 و به توان مثبت یا منفی بینهایت، جواب: حد جواب ندارد
9- عدد کوچکتر از منفی 1 باشد و به توان مثبت بینهایت، جواب: حد جواب ندارد
10- عدد کوچکتر از منفی 1 باشد و به توان منفی بینهایت، جواب: صفر

امیدوارم که اشتباه نکرده باشم!

سوال 4 تون هم مثل همین سوال، چندین حالت مختلف داره که فکر نمیکنم نوشتنش لزومی داشته باشه.

بقیه سوالات رو هم به نظرم دوستان درست پاسخ دادن.

1.عدد یا صفر حدی به روی صفر مطلق مبهمه؟ یا بی نهایت میشه؟
جواب ندارد(و در نتیجه حد وجود ندارد) البته شاید جواب حقیقی نداره؟
یه مثال: x=2/0 آیا این معادله ریشه داره؟ نه، چون ریشه ی این معادله باید عددی باشه
که وقتی در 0 ضرب میشه، جوابش بشه 2، و در میان اعداد حقیقی همچین عددی وجود نداره

3.یک به روی منفی بی نهایت چی میشه؟
صفر

Julian
09-06-2008, 15:31
سلام
چند تا سوال از حد داشتم اگه کسی جواب بده ممنون می شم
1.عدد یا صفر حدی به روی صفر مطلق مبهمه؟ یا بی نهایت میشه؟
2.عدد به توان مثبت بی نهایت چی میشه؟ عدد به توان منفی بی نهایت چطور؟
3.یک به روی منفی بی نهایت چی میشه؟
4.منفی بی نهایت و مثبت بی نهایت به توان عدد؟؟
5.فقط توابعی که کسری هستند می توانند مجانب قائم داشته باشند؟ اگر نه چه جوری میشه مجانبش رو پیدا کرد؟مثلا: 0=y3 x2 +y2 x +6x3 –y3 +2x-1
6.برای به دست آوردن مجانب قائم میشه اول کسر رو ساده کرد و بعد مجانب کسر ساده شده رو به دست آورد؟
همین!!
بازم ممنون. فعلا...

ریاضی؟ متنفرم.
ااخ مغزم تیر کشید.
قلبم وایساد.

eliass_fiend
11-06-2008, 12:37
سلام
چند تا سوال از حد داشتم اگه کسی جواب بده ممنون می شم
1.عدد یا صفر حدی به روی صفر مطلق مبهمه؟ یا بی نهایت میشه؟
2.عدد به توان مثبت بی نهایت چی میشه؟ عدد به توان منفی بی نهایت چطور؟
3.یک به روی منفی بی نهایت چی میشه؟
4.منفی بی نهایت و مثبت بی نهایت به توان عدد؟؟
5.فقط توابعی که کسری هستند می توانند مجانب قائم داشته باشند؟ اگر نه چه جوری میشه مجانبش رو پیدا کرد؟مثلا: 0=y3 x2 +y2 x +6x3 –y3 +2x-1
6.برای به دست آوردن مجانب قائم میشه اول کسر رو ساده کرد و بعد مجانب کسر ساده شده رو به دست آورد؟
همین!!
بازم ممنون. فعلا...

1)مبهم میشه
2)در دو حالت فکر کنم مبهم میشه
3)مبهم
6)اره حا لا اگر ریشه صورت در مخرج بود (ریشه مخرج )اونوقت دیگه مجانب قائم نیست

smohs
14-06-2008, 23:22
سلام...
ببخشيد ولي اگه ممكنه سريع جواب بديد(در عرض حد اكثر دو روز):
جواب هاي غير صفر دستگاه چهار معادله چهار مجهولي زير چي ميشه؟
تكرار مي كنم جواب هاي غير صفر:

7a – b = 0

4a + 11b = 0

7c – a – d = 0

4c + 11d – b = 0
البته اگه جواب دو تاي اولش رو هم پيدا كرديد ممنون ميشم بيان كنيد
ضمن اين كه اگه روش پيدا كردن چنين جواب هايي رو هم مي دونيد لطف كنيد بگيد
خيلي ممنون

sherlockholmz
15-06-2008, 08:06
سلام...
ببخشيد ولي اگه ممكنه سريع جواب بديد(در عرض حد اكثر دو روز):
جواب هاي غير صفر دستگاه چهار معادله چهار مجهولي زير چي ميشه؟
تكرار مي كنم جواب هاي غير صفر:

7a – b = 0

4a + 11b = 0

7c – a – d = 0

4c + 11d – b = 0
البته اگه جواب دو تاي اولش رو هم پيدا كرديد ممنون ميشم بيان كنيد
ضمن اين كه اگه روش پيدا كردن چنين جواب هايي رو هم مي دونيد لطف كنيد بگيد
خيلي ممنون

باسلام،
معادلات همگن (معادلاتي كه طرف ثابتشان صفر است) تنها در صورتي جواب غير صفر دارند كه دترمينان ضرائبشان صفر باشد.با توجه به دستگاه داده شده مي بينيم كه دو معادله اول مستقل از c و d هستند. پس ميتوان آنها را بصورت دستگاه دو معادله دو مجهول حل كرد وسپس جوابها را در دو معادله ديگر قرار داد و حل را كامل كرد:


[ برای مشاهده لینک ، لطفا با نام کاربری خود وارد شوید یا ثبت نام کنید ]


پس اين دستگاه همگن جوابي به غير از صفر ندارد.
مستقيما" هم مي توانستي دترمينان 4*4 ضرائب را حساب كني (كه ميشود 81*81 ) و چون اين دترمينان صفر نيست،معادله جوابي به غير از صفرنخواهد داشت.
موفق باشي

smohs
16-06-2008, 05:41
باسلام،
معادلات همگن (معادلاتي كه طرف ثابتشان صفر است) تنها در صورتي جواب غير صفر دارند كه دترمينان ضرائبشان صفر باشد.با توجه به دستگاه داده شده مي بينيم كه دو معادله اول مستقل از c و d هستند. پس ميتوان آنها را بصورت دستگاه دو معادله دو مجهول حل كرد وسپس جوابها را در دو معادله ديگر قرار داد و حل را كامل كرد:


[ برای مشاهده لینک ، لطفا با نام کاربری خود وارد شوید یا ثبت نام کنید ]


پس اين دستگاه همگن جوابي به غير از صفر ندارد.
مستقيما" هم مي توانستي دترمينان 4*4 ضرائب را حساب كني (كه ميشود 81*81 ) و چون اين دترمينان صفر نيست،معادله جوابي به غير از صفرنخواهد داشت.
موفق باشي



خيلي ممنون جناب هولمز عزيز
راستش خودمم مونده بودم چرا حل نميشه
البته توي يه مبحث معادلات ديفرانسيل بهش برخوردم
چندين بار هم چك كردم شايد اين معادله رو اشتباه بدست آورده باشم ديدم نه... درسته
احتمالا يه جاي كارم ايراد داشته
در هر حال ممنون از لطفتون:10:

Luxe
16-06-2008, 20:09
این انتگرال با چه روشی حل میشه میشه کمک کنید؟؟؟

[ برای مشاهده لینک ، لطفا با نام کاربری خود وارد شوید یا ثبت نام کنید ]

zahedy2006
17-06-2008, 00:11
این انتگرال با چه روشی حل میشه میشه کمک کنید؟؟؟

با این
[ برای مشاهده لینک ، لطفا با نام کاربری خود وارد شوید یا ثبت نام کنید ]
و اگه روش این رو بدونید با این هم باید بشه (تغییر متغیر و غیره)
[ برای مشاهده لینک ، لطفا با نام کاربری خود وارد شوید یا ثبت نام کنید ]
فکر کنم اگه t=1/x هم بگیری حل شه

sherlockholmz
17-06-2008, 10:51
این انتگرال با چه روشی حل میشه میشه کمک کنید؟؟؟

[ برای مشاهده لینک ، لطفا با نام کاربری خود وارد شوید یا ثبت نام کنید ]




[ برای مشاهده لینک ، لطفا با نام کاربری خود وارد شوید یا ثبت نام کنید ]

korsakov
21-06-2008, 13:45
این شخص چه مدارسی و چه آموزشگاه هایی تدریس کرده ؟

shiraz_670
23-06-2008, 15:59
سلام عزیزان هر چی سوال ریاضی داشتید اینجا بزارید:
[ برای مشاهده لینک ، لطفا با نام کاربری خود وارد شوید یا ثبت نام کنید ]

مکارم
27-06-2008, 08:14
سلام عزیزان
یک سؤالیه که مدتها هست که روش فکر کرده ام و نتونستم اونو حل کنم
یه حد ناقابله


[ برای مشاهده لینک ، لطفا با نام کاربری خود وارد شوید یا ثبت نام کنید ]{n\rightarrow\infty} \Pi(1-\frac{1}{2^k})=

[ برای مشاهده لینک ، لطفا با نام کاربری خود وارد شوید یا ثبت نام کنید ](1-\frac{1}{2})(1-\frac{1}{4})(1-\frac{1}{8})(1-\frac{1}{16})\dots(1-\frac{1}{2^n})\dots

shape
01-07-2008, 18:49
سلام
من تازه عضو شدم
سوال جالبیه

shape
07-07-2008, 01:53
فکر کنم که جواب جموجوری نداره
یه سر به لینک زیر بزن

[ برای مشاهده لینک ، لطفا با نام کاربری خود وارد شوید یا ثبت نام کنید ]

مکارم
07-07-2008, 09:21
سلام
دستت درد نکنه. لا اقل از ناراحتی نجاتم دادی !

farbod123
07-07-2008, 18:08
کسی جواب این سوال رو می دونه؟؟؟همه ی قطعات یکسانند ولی مساحت پایینی کمتره...:40:
[ برای مشاهده لینک ، لطفا با نام کاربری خود وارد شوید یا ثبت نام کنید ] and Settings\FARBOD\My Documents\kj.bmp

Meisam Khan
08-07-2008, 09:47
کسی جواب این سوال رو می دونه؟؟؟همه ی قطعات یکسانند ولی مساحت پایینی کمتره...[ برای مشاهده لینک ، لطفا با نام کاربری خود وارد شوید یا ثبت نام کنید ](39).gif

دوباره اپلود كن دوست عزيز

sherlockholmz
08-07-2008, 11:19
کسی جواب این سوال رو می دونه؟؟؟همه ی قطعات یکسانند ولی مساحت پایینی کمتره...:40:
[ برای مشاهده لینک ، لطفا با نام کاربری خود وارد شوید یا ثبت نام کنید ] cuments%5Ckj.bmp

چي؟كجا؟كي؟
من نمي بينم يا...؟

farbod123
08-07-2008, 13:12
دوباره اپلود كن دوست عزيز
اپلود کردم تو پرشین گیگ بفرما:::

برای مشاهده محتوا ، لطفا وارد شوید یا ثبت نام کنید

shape
08-07-2008, 16:57
زاویه ها ی کوچک مثلث های سبز و قرمز را در نظر بگیر.تانژانت هایشان 3/8 و 2/5 است .پس دو زاویه برابر نیستند.در اصل وتر یک خط شکسته است.

shape
08-07-2008, 17:23
دوست عزیز لازمه بدونی که اضلاع طوری انتخاب شدن که اشتباه کنی اگر کمی به اضلاع مجانب زاویه قائمه نیگا بندازی میفهمی که به ترتیب (2و5)و(3و8)و(5و13)هست.تو رو یاد چیزی نمیندازه......سری فیبوناتچی
در اصل اضلاع به صورت زیر انتخاب شدن:
[ برای مشاهده لینک ، لطفا با نام کاربری خود وارد شوید یا ثبت نام کنید ]
حالا با توجه به حالت خاص اتحاد کاتالان داریم:
[ برای مشاهده لینک ، لطفا با نام کاربری خود وارد شوید یا ثبت نام کنید ]
و همچنین:
[ برای مشاهده لینک ، لطفا با نام کاربری خود وارد شوید یا ثبت نام کنید ]
حالا فهمیدی چرا درست یه خونه کم می اوردی؟
یعنی این شیب ها نزدیک به هم هستن ولی هرگز برابر نمیشن
امیدوارم جوابتو خوب داده باشم :دی

farbod123
08-07-2008, 19:16
مرسی فهمیدم...

shape
09-07-2008, 21:51
یه سوال جالب که خودم حلش کردم.دنبال راه حل دیکه ای هستم .

سوال اینه:از بین دو عدد [ برای مشاهده لینک ، لطفا با نام کاربری خود وارد شوید یا ثبت نام کنید ] و [ برای مشاهده لینک ، لطفا با نام کاربری خود وارد شوید یا ثبت نام کنید ] کدوم بزرگتره.بدون کمک ماشین حساب.

miriam
12-07-2008, 12:06
این شخص چه مدارسی و چه آموزشگاه هایی تدریس کرده ؟

:5:who is he?

Luxe
16-07-2008, 16:22
سلام به اساتید
میشه درباره استقلال خطی و وابستگی (اشتباه که نگفتم) مثالیتوضیح بدین اگه منبعی کتابی پی دی اف باشه خیلی خوبه

من به هیچ منبعی الان دسترسی ندارم

shape
19-07-2008, 20:08
چه بد , کسی جواب نداد

7- اگر مساله ای را که خودتان روش حل آنرا می دانید مطرح کردید و کسی آن مساله را حل نکرد، برای غنی تر شدن مطالب انجمن، خواهشمندیم روش حل خود را حداقل به طور خلاصه بیان کنید.
تابع ریشه ی x ام x در e ماکسیمم میشود....

mohandesirani
24-07-2008, 10:38
آقا این سوالو چجوری باید حل کرد؟

مستطیل های محاط در یک دایره به قطر 6 واحد راحول یک ضلع خود دوران می دهیم تا استوانه های قائم ایجاد شود.
وقتی حجم این استوانه ها بیشترین مقدارادارد ارتفاع آن کدام است؟
4
دو رادیکال 3
2رادیکال6
3رادیکال 2

soheilsmart
24-07-2008, 10:57
اگه اشتباه نکنم این سوال کنکور امسال رشته ریاضیه!
ببخشید من نرم افزار ندارم واسه همین راه حل رو میگم!

قطر مستطیل محاط در دایره از مرکز دایره عبور میکند!(چون هر زاویه مستطیل 90 درجه اس (محاطی)و کمان روبرو به این زاویه باید 180 درجه باشه)
از رابطه فیثاغورث استفاده میکنیم!(یه ضلع مستطیل رو h فرض کن اون یکی همr
h^2+r^2=36

حالا رابطه حجم استوانه رو بنویس
و از بهینه سازی استفاده کن!
و در نهایت مشتق بگیر ومساوی صفر قرار بده!جواب بدست میاد

موفق باشی!

Animators
29-07-2008, 01:42
چگونگی حل معادله (X به توان 2 = Y)؟ باتشکر Animators :20:

mohammad_mgh
29-07-2008, 03:27
لطفا بگویید که این اعداد حروف چه معنایی دارند و چگونه خوانده می شوند
مثلا
1.7e +8
یا
3.5e -9

mohsen_blid
29-07-2008, 07:33
سلام دوستان خسته نباشید
سوال مهمی که داشتم اینه که ضد مشتق چیست؟ کسی هست اینجا اموزش بده چون نمیتونم یاد بگیرم و چند روز دیگه امتحان دارم لطفا کمک کنید

dkhatibi
29-07-2008, 10:09
ضد مشتق چیست؟
چیز عجیبی نیست! همون انتگراله!
مثلا می خواهید ضد مشتق تابع f را پیدا کنید، منظور تابعی است که اگر ازش مشتق بگیرید می شه همون تابع f.

mohsen_blid
29-07-2008, 10:29
فرمولی هست که بشه به راحتی جواب رو بدست اورد
یک نمونه میشه برام مثال بزنید

zahedy2006
29-07-2008, 11:51
فرمولی هست که بشه به راحتی جواب رو بدست اورد
یک نمونه میشه برام مثال بزنید


خيلي از فرمول هاي پر كاربردش براي شروع در كتاب ماشاء الله رضوي هست

مثلا x^n انتگرالش ميشه x^n+1/n+1 . از اون عبارت دوم مشتق بگيريد بر مي گرده به اولي

mohsen_blid
29-07-2008, 13:50
اگر میشه یک pdf بهم بدید که بتونم راحت تر انتگرال رو یاد بگیرم اگر فرمول های دیگر با تمرین و حلش رو دارید ممنون میشم برام بزارید
در ضمن برای برنامه های هذلولی و بیضی و ... فرمول های ساده رو دارید بهم بدید

khatarat
29-07-2008, 13:56
ما دو نقطه در دستگاه مختصات سه بعدی داریم.نقطه دوم رو حول نقطه اول در محور های x و y بصورت تصادفی می چرخانیم.حالا ما سه نقطه داریم که زاویه X , Y بین این سه نقطه رو می خوایم.به صورتی که وقتی این زوایا رو بدست آوردیم وقتی که نقطه سوم رو به اندازه Y- در جه و سپس آن را به اندازه X- در چرخانیم نقطه سوم دقیقا روی نقطه دوم قرار بگیره.
من نیاز فوری به جواب این مسئله دارم.

روح سگ ولگرد
30-07-2008, 11:47
من نفهمیدم نقطه سوم چجوری تشکیل شد؟

Maxwell_1989
30-07-2008, 13:06
ببین دوست عزیز شما فرمولهای مشتق رو که بلدی.توی ستون اول یه جدول،توابعو بنویس توی ستون روبروش مشتقشونو. حالا برای بدست اوردن فرمولهای انتگرال جای دو ستونو با هم عوض می کنیم.یعنی ستون دوم جدول قبلی میشه تابع و ستون اولش میشه انتگرال اون تابع.
در مورد بحث مقاطع مخروطی هم فرمول یاد گرفتن هیچ دردی رو دوا نمیکنه .باید مطلبو یاد بگیری .چون مثلا توی امتحان ازت نمی پرسن معادله ی بیضی چیه؟!
با تشکر.

mohsen_blid
30-07-2008, 14:43
انتگرال معین رو تمام کردم و جالب بود
اما یه سوال
ضد مشتق همون انتگرال نامعین است؟
عزیز من تمام هذلولی بیضی و غیره رو بلدم اما نیاز به یک فرمول ساده و سریع دارم تا زودتر به جواب برسم سر امتحان وقت کم نیارم

khatarat
30-07-2008, 17:39
ببین از چرخش نقطه دوم حول نقطه اول یه نقطه جدید داریم.
یعنی نقطه اول مرکز است.نقطه دوم هم که معلومه و نقطه سوم حاصل چرخش نقطه دوم به دور مرکز است.

mohsen_blid
30-07-2008, 19:12
سلام
اگر براتون مقدور است برای من یک جزوه خوب برای ریاضی یک پیام نور بده چون این کتاب خیلی بد توضیح داده
اگر میتونید جزوه ای برای انتگرال ناسره به من بدید چون اصلا نمیدونم چطوری باید شروع کرد وقتم هم زیاد نیست
حداقل ممکن این جزوه در حد عامیانه باشه یعنی راحت بشه مفهوم رو گرفت و هی تابع در تابع نگفته باشد(زبان ریاضی نباشه)

Maxwell_1989
31-07-2008, 00:06
شما اول بفرمایید چه کتابی رو میخونید.بعد من کتاب یه استاد دیگه که بهتر توضیح داده باشه رو براتون میذارم.چون برای ریاضی 1 پیام نور چند کتاب موجود هست.-با تشکر

mohsen_blid
31-07-2008, 06:47
درسته من کتاب ریاضی 1 عمومی نوشته ابراهیم احمد پور و انه گلدی مهمیانی رو دارم
گر بتونید کمکم کنید یه عمر دعات میکنم

Maxwell_1989
31-07-2008, 15:11
والا یه جزوه ریاضی1 هست مال استاد شهشهانی از دانشگاه شریف.نمیدونم چقدر به دردت بخوره:(از پیام نور چیزی نتونستم گیر بیارم.)

برای مشاهده محتوا ، لطفا وارد شوید یا ثبت نام کنید

mohsen_blid
31-07-2008, 16:42
ممنون ولی این نمیتونه کمک کنه چون مطالبی که من می خوام رو نداره تازه این خیلی حجم هم داره
اگر میتونی تو همین بخش ریاضیات من یک تاپیک باز کردم به نام فرمول های محاسبه اگر بتونی به من در اونها کمک کنی خیلی کارم جلو میره

Arsa Persian
01-08-2008, 18:42
چگونگی حل معادله (X به توان 2 = Y)؟ باتشکر Animators :20:

اگر X به توان 2 منظور باشد که راحت هستش از طرفین رادیکال میگیریم و بعد جواب بدست میاد
اگر 2 به توان X منظور باشد از طرفین لگاریتم در مبنای 2 میگیریم داریم:

Log Y = Ln X^2
Log Y = X

در نتیجه : X = Log Y
حال برای اینکه بگیم معادله باید Y یک عدد باشه که در اون صورت X جواب معدله است
اگر Y نداده شده باشد X جواب کلی معاده است
*توضیح: ببخشید که مبنای 2 لگاریتم رو ننوشتیم P30World از این امکانات نداره

خوش باشید:20:

Armin 2008
02-08-2008, 20:32
[ برای مشاهده لینک ، لطفا با نام کاربری خود وارد شوید یا ثبت نام کنید ]

Maxwell_1989
02-08-2008, 22:17
الف)سی یک+سی دو+ایکس مساوی 180
ب)سی یک+سی دو+آ+دی مساوی 180
ج)بی یک+سی یک+آ مساوی 180
د)بی دو+سی دو+دی مساوی 180
ه)سی یک=آ
و)سی دو=دی
توی الف و ب و ج و د،ه و "و" رو جایگزین کن.
بعد ج و د رو جمع کن پس آ+دی=90
پس از ب:سی یک+سی دو=90
در نهایت از الف:x=90
متشکرم

mohsen_blid
03-08-2008, 16:01
سلام دوست من
خسته نباشید
فقط برای اطمینان می پرسم
ضد مشتق همان انتگرال است که دارای 10فرمول است درسته؟فرمولاش که بیشتر نیست؟

mohsen_blid
03-08-2008, 16:08
ممنون میشم اگر نوع فرمول و همچنین راه حل این 2 سوال را در عکس بگید
[ برای مشاهده لینک ، لطفا با نام کاربری خود وارد شوید یا ثبت نام کنید ]

mohsen_blid
04-08-2008, 10:24
منتظر جواب دوستان هستم

zahedy2006
04-08-2008, 11:34
مورد ب كه چيزي نداره. اول 2/1 را كشيده بيرون
بعد صورت و مخرج را ساده كرده
بعد انتگرال راديكال ايكس +1 را گرفته

مورد ج از تغيير متغير استفاده كرده است. يعني زير راديكال را برابر u^2 گرفته و du و dx را ساخته و جايش قرار داده
مورد د هم عين بالا

mohsen_blid
04-08-2008, 16:33
یعنی چی ساده کرده بعد اون 3/2 رو از کجا اورده بعد من فرمول انتگرال رادیکالی ندارم میشه فرمولش رو نشون بدید
چجوری du و dx ساخته من هر چی چپ راستش میکنم نمیگیرم بخاطر پیری زودرسه فرمولش چجوری است
تا بعدا برسیم به د

Maxwell_1989
04-08-2008, 21:53
سلام!
شما میدونید که رادیکال فلان یعنی فلان به توان یک دوم.پس انتگرال رادیکالی فرمولش میشه مثل همون انتگرال یو بتوان ان.اون فرمولا که توی تاپیک فرمولهای ریاضی مهم گذاشتم خوندی؟
در مورد سوال دوم هم:u رو که بگیری مساوی ... از فرمول "دیفرانسیل یو=یوپریم در دیفرانسیل ایکس"استفاده می کنی و جا گذاری می کنی.
من فکر می کنم توی انتگرال باید خیلی مساله حل کنی تا مسلط بشی.بهت کتاب دیفرانسیل و انتگرال نامعین از محمد عابدی انتشارات مدرسه رو پیشنهاد می کنم.توپ توپت می کنه.
با تشکر

Armin 2008
07-08-2008, 12:35
[ برای مشاهده لینک ، لطفا با نام کاربری خود وارد شوید یا ثبت نام کنید ]

Armin 2008
07-08-2008, 12:42
[ برای مشاهده لینک ، لطفا با نام کاربری خود وارد شوید یا ثبت نام کنید ]

shape
07-08-2008, 14:53
برا سوال اولت یه استقرای ساده روی تعداد عوامل اول میزنیم

برای حالت [ برای مشاهده لینک ، لطفا با نام کاربری خود وارد شوید یا ثبت نام کنید ] مقسوم علیه ها [ برای مشاهده لینک ، لطفا با نام کاربری خود وارد شوید یا ثبت نام کنید ] هستن که داریم:

[ برای مشاهده لینک ، لطفا با نام کاربری خود وارد شوید یا ثبت نام کنید ]

فرض کنید برا [ برای مشاهده لینک ، لطفا با نام کاربری خود وارد شوید یا ثبت نام کنید ] ثابت کردیم.برا حالت [ برای مشاهده لینک ، لطفا با نام کاربری خود وارد شوید یا ثبت نام کنید ] اینطوری عمل

میکنیم.
مقسوم علیه های [ برای مشاهده لینک ، لطفا با نام کاربری خود وارد شوید یا ثبت نام کنید ] به ترتیب برابرند با:
*.مقسوم علیه های

*. [ برای مشاهده لینک ، لطفا با نام کاربری خود وارد شوید یا ثبت نام کنید ] ضرب در مقسوم علیه های [ برای مشاهده لینک ، لطفا با نام کاربری خود وارد شوید یا ثبت نام کنید ]
.
.
.

* [ برای مشاهده لینک ، لطفا با نام کاربری خود وارد شوید یا ثبت نام کنید ] ضرب در مقسوم علیه های [ برای مشاهده لینک ، لطفا با نام کاربری خود وارد شوید یا ثبت نام کنید ]

پس داریم:

[ برای مشاهده لینک ، لطفا با نام کاربری خود وارد شوید یا ثبت نام کنید ]

که [ برای مشاهده لینک ، لطفا با نام کاربری خود وارد شوید یا ثبت نام کنید ] نشون دهنده ی جمع مقسوم علیه های [ برای مشاهده لینک ، لطفا با نام کاربری خود وارد شوید یا ثبت نام کنید ] هست.

سوال دومت هم اینجوری حل میشه

اگه [ برای مشاهده لینک ، لطفا با نام کاربری خود وارد شوید یا ثبت نام کنید ] یه مقسوم علیه باشه [ برای مشاهده لینک ، لطفا با نام کاربری خود وارد شوید یا ثبت نام کنید ] هم یه مقسوم علیه هست که ضرب این دو تا میشه [ برای مشاهده لینک ، لطفا با نام کاربری خود وارد شوید یا ثبت نام کنید ]

پس ضرب [ برای مشاهده لینک ، لطفا با نام کاربری خود وارد شوید یا ثبت نام کنید ] تا مقسوم علیه میشه [ برای مشاهده لینک ، لطفا با نام کاربری خود وارد شوید یا ثبت نام کنید ]
چون هر دو تا مقسوم علیه یه [ برای مشاهده لینک ، لطفا با نام کاربری خود وارد شوید یا ثبت نام کنید ] تولید میکنه.

sonkia
08-08-2008, 13:37
من ds اشکال مختلف هندسی رو میخوام مثل مثلث و دایره و ...
برای المان گیری نیازشون دارم!
توروخدا فورا جواب بدید !!!!!!

shape
08-08-2008, 19:44
کره:
[ برای مشاهده لینک ، لطفا با نام کاربری خود وارد شوید یا ثبت نام کنید ]
دایره:
[ برای مشاهده لینک ، لطفا با نام کاربری خود وارد شوید یا ثبت نام کنید ]
مثلث:
[ برای مشاهده لینک ، لطفا با نام کاربری خود وارد شوید یا ثبت نام کنید ]
مستطیل:
[ برای مشاهده لینک ، لطفا با نام کاربری خود وارد شوید یا ثبت نام کنید ]
مخروط با مولد L و شعاع r:
[ برای مشاهده لینک ، لطفا با نام کاربری خود وارد شوید یا ثبت نام کنید ]
استوانه:
[ برای مشاهده لینک ، لطفا با نام کاربری خود وارد شوید یا ثبت نام کنید ]
بیضی:
[ برای مشاهده لینک ، لطفا با نام کاربری خود وارد شوید یا ثبت نام کنید ]

Parnyan
16-08-2008, 00:28
سلام

میشه یکی لطف کنه جوابه این سوالارو با راه حل بزاره ....ممنون میشم .... ( تا 2شنبه میخوامشون )

[ برای مشاهده لینک ، لطفا با نام کاربری خود وارد شوید یا ثبت نام کنید ]


AlL ThE Best

Parnyan
18-08-2008, 12:19
اینجا هیشکی نییییییییییییییست ؟؟؟؟؟؟:41::13::19:

soheilsmart
19-08-2008, 17:47
سلام!
این جواب قسمت اول سوالتون!

برای مشاهده محتوا ، لطفا وارد شوید یا ثبت نام کنید
فقط راه حلو خلاصه نوشتم!
اگه ابهامی بود بگید!

soheilsmart
19-08-2008, 18:01
این هم قسمت دوم!

برای مشاهده محتوا ، لطفا وارد شوید یا ثبت نام کنید

این هم قسمت سوم!


برای مشاهده محتوا ، لطفا وارد شوید یا ثبت نام کنید

فقط امیدوارم جایی اشتباه نکرده نباشم!

Parnyan
19-08-2008, 20:59
این هم قسمت دوم!

برای مشاهده محتوا ، لطفا وارد شوید یا ثبت نام کنید

این هم قسمت سوم!


برای مشاهده محتوا ، لطفا وارد شوید یا ثبت نام کنید

فقط امیدوارم جایی اشتباه نکرده نباشم!

سلام

دستتون درد نکنه ....

میشه لطف کنین بگین چرا تو سوال 3 گفتین x^2-5x کوچیکتر مساوی 1 ؟؟؟؟:41:

soheilsmart
19-08-2008, 22:10
سلام

دستتون درد نکنه ....

میشه لطف کنین بگین چرا تو سوال 3 گفتین x^2-5x کوچیکتر مساوی 1 ؟؟؟؟:41:
بله!
لگاریتم چون زیر رادیکال هستش
باید بزرگ تر مساوی صفر باشه
چون مبنا 1/2 =0.5 هست ( بین صفر و یک) جهت نامساوی عوض میشه!(اعداد بین صفر و یک اگر به توان مثبت برسند یا صفر کوچکتر مساوی یک و بزرگتر از صفر میشوند!)
1/2 به توان صفر میشه 1!

Parnyan
26-08-2008, 19:05
سلام

میشه یکی لطف کنه این چند تا سوالو برام حل کنه با راه حل ؟؟؟

[ برای مشاهده لینک ، لطفا با نام کاربری خود وارد شوید یا ثبت نام کنید ]

Parnyan
26-08-2008, 23:50
سلام

2 تا سواله دیگه ....اگه لطف کنین جواب و راه حل رو بنویسین برام ممنون میشم


[ برای مشاهده لینک ، لطفا با نام کاربری خود وارد شوید یا ثبت نام کنید ]

sherlockholmz
27-08-2008, 15:56
باسلام


[ برای مشاهده لینک ، لطفا با نام کاربری خود وارد شوید یا ثبت نام کنید ]


موفق باشيد.

sherlockholmz
27-08-2008, 16:08
بادر نظر گرفتن اطلاعات دبيرستاني:

[ برای مشاهده لینک ، لطفا با نام کاربری خود وارد شوید یا ثبت نام کنید ]

پس جواب شما صحيح است.
موفق باشيد.

sherlockholmz
27-08-2008, 16:12
[ برای مشاهده لینک ، لطفا با نام کاربری خود وارد شوید یا ثبت نام کنید ]


موفق باشيد.

sherlockholmz
27-08-2008, 16:22
چون ما از طرف منفي به يك نزديك ميشويم، پس داخل جزء صحيح عددي بين 1- و 0 است و در نتيجه جزء صحيح آن 1- است.پس حد وجود دارد و مقدار آن نيز صفر است.(توجه كنيد كه حد چپ و راست مخرج يكسان نيست اما هردو وجود دارند و چون حد صورت صفر است، حد چپ و راست تابع برابر با هم و صفراست ،پس تابع در نقطه 1 حد دارد.
موفق باشيد.

sherlockholmz
27-08-2008, 16:35
[ برای مشاهده لینک ، لطفا با نام کاربری خود وارد شوید یا ثبت نام کنید ]

هردو هم قابل قبول است.
موفق باشيد.

MRTZ HSV
07-09-2008, 17:11
اگر منحني به معادله y=2x^2-4x+m-3 محور x ها را در نقاط مثبت قطع كند آنگاه مجموع مقادير M چه خواهد بود؟

Maxwell_1989
07-09-2008, 20:44
برای ایجاد چنین شرایطی a باید بزرگتر از 0 باشد که هست.دلتا را باید منفی بگذارید.چون در این صورت است که"همواره ریشه ها مثبت خواهند شد"

MRTZ HSV
07-09-2008, 22:11
ببخشيد من سال اولم
مي خوام برم امسال 2 رياضي خيلي هم به اين جور مسائل علاقه دارم

اين تست كنكور همين امسال بود 87 اولين تست هم بود
من حلش كردم و به جواب درست هم رسيدم ولي مي خواستم ببينيم شما از چه راه هايي استفاده ميكنيد

Maxwell_1989
08-09-2008, 06:25
اگه تونستید با روشی غیر از اونی که من گفتم و با استفاده از معلومات اول دبیرستان حلش کنید بهتون تبریک میگم!

MRTZ HSV
08-09-2008, 11:11
Maxwell_1989 ([ برای مشاهده لینک ، لطفا با نام کاربری خود وارد شوید یا ثبت نام کنید ])
جان
شما راه كامل خودت رو برام بزار
منم راهم رو ميزارم

MRTZ HSV
08-09-2008, 13:34
بعيد مي دونم بقهميد چي كار كردم

[ برای مشاهده لینک ، لطفا با نام کاربری خود وارد شوید یا ثبت نام کنید ]

attractive_girl
11-09-2008, 15:08
سلام.لطفا هر کس جواب سوال زیر رو بلده تا روز دوشنبه به این سوال من پاسخ بده.
تابعی یک به یک و پوشا از مجموعه ی اعداد صحیح به مجموعه ی اعداد طبیعی بیابید.(ثابت کنبد اعداد صحیح و اعداد طبیعی هم ارز هستند)
mamnoon

attractive_girl
11-09-2008, 15:23
فقط در حد فصل اول پیش به پایین

Maxwell_1989
11-09-2008, 15:43
کافیه مجموعه اعداد طبیعی رو توی یک خط بنویسید صحیح ها رو تو خط پایین بعد هر عضو اولی رو به یک عضو دومی وصل کنید!
ولی جدا اینا تو تخصص بچه های آنالیزکاره مث دوست خوبم shape.از ایشون بپرسید.

chessmathter
11-09-2008, 20:15
سلام.لطفا هر کس جواب سوال زیر رو بلده تا روز دوشنبه به این سوال من پاسخ بده.
تابعی یک به یک و پوشا از مجموعه ی اعداد صحیح به مجموعه ی اعداد طبیعی بیابید.(ثابت کنبد اعداد صحیح و اعداد طبیعی هم ارز هستند)
mamnoon
حرف maxwell_1989 کاملا درسته ولی چون اعداد صحیح از 2 طرف نامحدوده گیج کنندست واسه همین اینو من پیشنهاد میکنم
اعداد صحیح منفی رو به مربعاتشون وصل میکنی

1-به 1
2-به 4
...

بعد اعداد صحیح نا منفی روتو یه خط و زیرش اعداد طبیعی به غیر از مربع کاملها رو مینویسیم به ترتیب وصل میکنیم به هم




.....,0,1,2,3,4,5

....,2,3,5,6,7,8

m1367m2006
11-09-2008, 20:54
دوست عزیز سلام اول این تعریف را داشته باش
ازنظره نموداری تابعی پوشاست که موقعی که خطوط افقی رسم می کنیم فقط نمودار را حداقل در یه نقطه قطع کنه! و تابع یک بیک حداکثر یه نقطه!
برهمین اساس یه تابع دو ضابطه ای مثه این F=x+1:X>=0و F=x^2:X<0 تعریف کن
یادت باشه چون میخواهی همتوانی اثبات کنی باید تابع بودن و پوشا ویک بیک بودن را جدا جدا اثبات کنی

chessmathter
11-09-2008, 21:45
دوست عزیز سلام اول این تعریف را داشته باش
ازنظره نموداری تابعی پوشاست که موقعی که خطوط افقی رسم می کنیم فقط نمودار را حداقل در یه نقطه قطع کنه! و تابع یک بیک حداکثر یه نقطه!
برهمین اساس یه تابع دو ضابطه ای مثه این F=x+1:X>=0و F=x^2:X<0 تعریف کن
یادت باشه چون میخواهی همتوانی اثبات کنی باید تابع بودن و پوشا ویک بیک بودن را جدا جدا اثبات کنی
بابا این که نخواست اثبات کنه گفته بیابید

بعد چیزی که نوشتی یک به یک نیست

x=-3----->y=9

x=8------->y=9

:46::20:

m1367m2006
11-09-2008, 23:06
بابا این که نخواست اثبات کنه گفته بیابید

بعد چیزی که نوشتی یک به یک نیست

x=-3----->y=9

x=8------->y=9

:46::20:




از شما ممنون بله من اشتباه جواب داده بودم که عذر خواهی میکنم من سوال را درست نفهمیده بودم الان داشتم چک می کردم که دیدم یک بیک نیست بازم ببخشید:31:

ثابت کنبد اعداد صحیح و اعداد طبیعی هم ارز هستند
دوست عزیز تا اونجا که یادمه وقتی می خواستن بگن یه مجموعه با اعداد طبیعی هم ارزه یا همتوانه از اعداد طبیعی به اون مجموعه تابع تعریف می کردن.
مثلا ما برای اثبات اینکه مجموعه اعداد صحیح شماراست(م:تاپیک بهشت کانتور) باید اثبات کنیم که این مجموعه با اعداد طبیعی همتوانه(همتوانی یه رابطه هم ارزی است)
پسF:N......>Z
nزوج:f=n/2
nفرد:f=n-1/2
موفق باشی................:8:

shape
11-09-2008, 23:22
به اینجا یه سر بزن

برای مشاهده محتوا ، لطفا وارد شوید یا ثبت نام کنید

chessmathter
12-09-2008, 00:24
دوست عزیز تا اونجا که یادمه وقتی می خواستن بگن یه مجموعه با اعداد طبیعی هم ارزه یا همتوانه از اعداد طبیعی به اون مجموعه تابع تعریف می کردن.
مثلا ما برای اثبات اینکه مجموعه اعداد صحیح شماراست(م:تاپیک بهشت کانتور) باید اثبات کنیم که این مجموعه با اعداد طبیعی همتوانه(همتوانی یه رابطه هم ارزی است)
پسF:N......>Z
nزوج:f=n/2
nفرد:f=n-1/2
موفق باشی................:8:
این attractive_girlیه تابع از z----->n خواست
حالا اگه موضوع اثبات هم ارزی اعداد صحیح وطبیعی باشه باید یه تناظر بین دو مجموعه ایجاد کنیم یعنی یه تابع پوشا و 1-1 ولی تابعی که نوشتین پوشا نیست یعنی اعدادمنفی و در بر نمیگیره.به نظر من با نوشتن ضابطه نمیشه جواب داد به جواب من وجواب shape نگاه کنید(البته من با اون جواب مشکل دارم یعنی اصلا نگرفتم چی میگه)
[quote]اعداد صحیح منفی رو به مربعاتشون وصل میکنی

1-به 1
2-به 4
...

بعد اعداد صحیح نا منفی روتو یه خط و زیرش اعداد طبیعی به غیر از مربع کاملها رو مینویسیم به ترتیب وصل میکنیم به هم




.....,0,1,2,3,4,5

....,2,3,5,6,7,8





quote/]

برای مشاهده محتوا ، لطفا وارد شوید یا ثبت نام کنید

m1367m2006
12-09-2008, 09:26
این attractive_girlیه تابع از z----->n خواست
حالا اگه موضوع اثبات هم ارزی اعداد صحیح وطبیعی باشه باید یه تناظر بین دو مجموعه ایجاد کنیم یعنی یه تابع پوشا و 1-1 ولی تابعی که نوشتین پوشا نیست یعنی اعدادمنفی و در بر نمیگیره.به نظر من با نوشتن ضابطه نمیشه جواب داد به جواب من وجواب shape نگاه کنید(البته من با اون جواب مشکل دارم یعنی اصلا نگرفتم چی میگه)
[quote]اعداد صحیح منفی رو به مربعاتشون وصل میکنی

1-به 1
2-به 4
...

بعد اعداد صحیح نا منفی روتو یه خط و زیرش اعداد طبیعی به غیر از مربع کاملها رو مینویسیم به ترتیب وصل میکنیم به هم




.....,0,1,2,3,4,5

....,2,3,5,6,7,8





quote/]

برای مشاهده محتوا ، لطفا وارد شوید یا ثبت نام کنید
دوست عزیز من با توجه به صورت سوال مجموعه اعداد طبیعی را دامنه قرار دادم درست شد.
در ضمن گفتم این سوال بحثی از نظریه مجموعه ها ست (تاپیک بهشت کانتور) که همان طور که دیدید shapeعزیز هم پستی از همون جا نقل قول کرده لطفن پست را دقیق بخون
البته اگه اصرار دامنه بودن اعداد صحیح باشه اینجا فک نکنم ضابطه جواب بده:41:

m1367m2006
12-09-2008, 10:39
اگه راه حل های ارائه شده را نفهمیدی اینا را ببین امید وارم بفهمی!
قضیه:گیریمAوBوCوD چهار مجموعه باشند به طوری که AباBهم ارز و اشتراک انها تهی است و Cبا Dهم ارز و اشتراک انها تهی است در این صورت اجتماع AباC همتوان است با اجتماعBباD
بر طبق این قضیه دو تابع اولی از اعداد صحیح مثبت (شامله صفر) به اعداد طبیعی فرد با ضابطه 2x+1 و دومی از اعداد صحیح منفی به اعداد طبیعی زوج با ضابطه 2x- تعریف میکنیم. طبق قضیه گفته شده اجتماع اعداد صحیح مثبت واعداد صحیح منفی که برابر است با اعداد صحیح با اجتماع اعداد طبیعی زوج و اعداد طبیعی فرد یعنی اعداد طبیعی هم ارز است
بهمین سادگی .فک کنم دیگه درست شد مگر اینکهchessmathter جان ایرادی بهش بگیره

chessmathter
12-09-2008, 11:02
اختیار دارین این یکی راه حل خیلی زیبا و ریاضیاتی بود!.:20:
من فقط میگم قضیه رو پچیده نکنین یه مثال خواست بیچاره!!
ولی در مورد پاسخ قبلی من با دامنه گرفتن N مشکل ندارم فقط میگم برد تابع پوشا نیست یعنی همه اعداد صحیح و نمیگیره

F:N......>Z
nزوج:f=n/2
nفرد:f=n-1/2
برد میشه{0,1,2,3,...} واصلا یه چیز جالب که الان دیدم پوشا هم نیست!!!:13:
(2,1)و(3,1)
ولی با این یکی حال کردم با حال بود:10:

chessmathter
12-09-2008, 11:11
این راه حل آخرکه m1367m2003 داد ساده وقشنگ بود فکر کنم همین واسه پاسخ سوال خوبه.

m1367m2006
12-09-2008, 11:54
برد میشه{0,1,2,3,...} واصلا یه چیز جالب که الان دیدم پوشا هم نیست!!![ برای مشاهده لینک ، لطفا با نام کاربری خود وارد شوید یا ثبت نام کنید ]
(2,1)و(3,1)
ولی با این یکی حال کردم با حال بود
اهان درسته ممنون فک کنم ماه رمضون رو مغزم اثر گذاشته شایدم چون ساعت 12شب بود این مورد راچک نکردم..........
یه چیزه دیگه دوست عزیز اگه این را می خواهی به استادی کسی نشون بدی فک کنم اثبات قضیه را هم ازت بخواد اگه خواستی بگو بذارم.

chessmathter
12-09-2008, 12:07
مطمعا خوشحال میشم البته نه برای استاد برای خودم
میخوام که بدونم

m1367m2006
12-09-2008, 13:05
مطمعا خوشحال میشم البته نه برای استاد برای خودم
میخوام که بدونم
چشم یه ذره پیچیدست برای همین 4 قسمتش کردم!
1. ازاینکه AباBهم ارزه و Cبا D نتیجه مگیریم که توابعی پوشا ویک بیک مانند f از A به B و g از C به D وجود داره
2. ما یه تابع مانند h از اجتماع AباCبه اجتماعBباD تعریف می کنیم که اگه ثابت کنیم یک بیک وپوشاست مشکل حله!
3.دو تا عضو از مجموعه h در نظر میگیریم مثه اینا (yوx1) و (yوx2) سه حالت بوجود میاد ( h همون اجتماع fوg است)
. اگه هر دو عضو متعلق به f بودن چون f یک بیک است کل تابع یک بیک.
..اگه هر دو عضو متعلق به g بودن چون g یک بیک است کل تابع یک بیک.
... اگه یه عضو متعلق به g ودیگری بهf چون اشتراک BوِِD تهی است غیر ممکن پس تلبع یک بیک است.
4.برای پوشا بودن یه عضو مانند y در نظر میگیریم که متعلق به اشتراک BوِِD است
اگه y متعلق به Bپس یه عضو مانندt در A وجود دارد که زوج مرتب yوt متعلق به h باشه پس تابع پوشا(چراشا اگه گفتی)
به همین ترتیب اگه y متعلق بهD باشه همین استدلال وجود داره
پس تابع پوشاست..[ برای مشاهده لینک ، لطفا با نام کاربری خود وارد شوید یا ثبت نام کنید ]

attractive_girl
12-09-2008, 13:29
بابا ایول به همتون .1 روزه جواب دادین.ایوا ایول.همه مخا اینجا جمعند.دست همتون درد نکنه.مرسسسسسسسسسسییییی

chessmathter
12-09-2008, 14:42
تعداد حالت هایی که میشه 8 تا وزیر تو خونه های شطرنج گذاشت که هیچ کدوم همدیگرو تهدید نکنن چند تاست؟
من خودم بعد از یه ایده سوال و به این convert !کردم
تعداد توابع از {1,2,3,4,5,6,7,8} به {1,2,3,4,5,6,7,8} که یک به یک و پوشاست و جمع مولفه اول و دوم و تفاضل مولفه دوم از مولفه اول هر زوج مرتبش متفاوت باشه؟
مثلا این یکیشه
{(1,4),(2,6),(3,1),(4,5),(5,2),(6,8),(7,3),(8,7)}
{15,10,14,7,9,4,8,5} جمع مولفه های هر زوج مرتب
{2,4,3-,3,1-,4,2-,1-} تفاضل مولفه دوم از مولفه اول
فعلا اینجا هنگ کردم [ برای مشاهده لینک ، لطفا با نام کاربری خود وارد شوید یا ثبت نام کنید ]

اگه کسی تونست بقیه ایده منو بره و حل کنه بگه اگه هم راه حل سوال و میدونه یا حل کرده بگه(سوال اصلی همون صفحه شطرنج ست)

AbdY_777
14-09-2008, 22:45
سلام دوستان
اين اولين مسئله اي هست كه من ارسال مي كنم اميد وارم خيلي زود بتونيد حلش كنيد ،

[ برای مشاهده لینک ، لطفا با نام کاربری خود وارد شوید یا ثبت نام کنید ]

سپاسگذارم

Antonio Andolini
21-09-2008, 03:55
اثبات قضیه ی فیثاغورث از طریق مساحت ذوزنقه رو می خواستم.
ممنونم

chessmathter
21-09-2008, 14:40
اینم اثبات قضیه با ذوزنقه به 2 روش!:

1.

[ برای مشاهده لینک ، لطفا با نام کاربری خود وارد شوید یا ثبت نام کنید ]

2.

[ برای مشاهده لینک ، لطفا با نام کاربری خود وارد شوید یا ثبت نام کنید ]


اینم مثل همون مساحت ذوزنقه رو با مساحت جمع سه قسمت رنگی مساوی بزاری وساده کنی به فرمول معروفمون میرسی.!!!
[ برای مشاهده لینک ، لطفا با نام کاربری خود وارد شوید یا ثبت نام کنید ]

Maxwell_1989
21-09-2008, 14:42
هر چی رابطه توی ذوزنقه و مثلث هست رو بنویسید به همراه شکل،با چند تا جاگذاری اثبات میشه.

Antonio Andolini
23-09-2008, 00:35
اینم اثبات قضیه با ذوزنقه به 2 روش!:




2.

[ برای مشاهده لینک ، لطفا با نام کاربری خود وارد شوید یا ثبت نام کنید ]


ا





باید مثلث قهوه ای رو قائم الزاویه بگیریم دیگه؟ .. چه جوری؟

chessmathter
23-09-2008, 13:53
باید مثلث قهوه ای رو قائم الزاویه بگیریم دیگه؟ .. چه جوری؟
اصلا لازم نیست بگیری که.!!! وقتی 2 تا مثلث قائم الزاویه رو اونطوری مثل شکل کنار هم بزاری زاویه وسط 90 درجه میشه .

peyman_nn
23-09-2008, 20:11
سلام

ببخشید یک روشی بود که در اون عددی را که بر 7 تقسیم پذیر بود رو پیدا میکردیم البته بدون تقسیم ، میدونید ؟

Maxwell_1989
24-09-2008, 01:22
برای یافتن باقیمانده تقسیم یک عدد بر 7 و 13،در آن عدد،سه رقم سه رقم از راست جدا می کنیم و متوالیا از هم کم و با هم جمع می کنیم(اعداد بدست آمده را).سپس عدد حاصل را بر 7 یا 13 تقسیم می کنیم.باقیمانده تقسیم بدست می آید(که اگر 0 باشد،بخشپذیری رو داریم)
مثلا3641892:
892-641+3=254
حالا باقیمانده 254 بر 7 رو بدست میاریم.میشه همون چیزی که میخواستیم.پس باقیمانده 3641892 در تقسیم بر 7،میشه2.

atlanta
28-09-2008, 20:02
سلام اگر امکان دارد لطفاً در مورد فصل اول ریاضیات 2 توضیحاتی ارائه دهید به خصوص قسمت اول آن .
با تشکر

attractive_girl
28-09-2008, 21:12
.سلام من سوالي داشتم براي برادرم که خيلي باهوشه ولي هرچي فکر مي کنه نميتواند آن را حل کند!راستش من خودمم حوصله اش رو نداشتم روش فکر کنم:27:
لطفا راه حلي در حد کلاس دوم راهنمايي بدهيد
متن سوال اين است:

مجموعه اي 12 عضو دارد تعداد زير مجموعه هاي 4 عضوي آن چقدر است؟
ممنون

zahedy2006
28-09-2008, 21:26
در حد دوم راهنمایی که میشه نوشتن مجموعه ها ولی با ترکیبیات میشه c(12,4)

chessmathter
28-09-2008, 21:42
سلام اگر امکان دارد لطفاً در مورد فصل اول ریاضیات 2 توضیحاتی ارائه دهید به خصوص قسمت اول آن .
با تشکر
میشود موضوع فصل را بگویید؟!آخه من جه میدونم فصل اول ریاضیات 2 چیه !حفظ که نمکنیم!!!

chessmathter
28-09-2008, 22:41
.سلام من سوالي داشتم براي برادرم که خيلي باهوشه ولي هرچي فکر مي کنه نميتواند آن را حل کند!راستش من خودمم حوصله اش رو نداشتم روش فکر کنم:27:
لطفا راه حلي در حد کلاس دوم راهنمايي بدهيد
متن سوال اين است:

مجموعه اي 12 عضو دارد تعداد زير مجموعه هاي 4 عضوي آن چقدر است؟
ممنون


در حد دوم راهنمایی که میشه نوشتن مجموعه ها ولی با ترکیبیات میشه c(12,4)
حالا ببین اینطوری میتونی توضیح بدی؟.
برای عضو اول 12 انتخاب است برای دومی 11 تا ... پس تعداد حالتها میشه 12*11*10*9
ولی چون تویه مجموعه ترتیب مهم نیست مثلا {a,b,c,d}و {a,c,d,b} یکیه پس تعداد مجموعه های تکراری با a,b,c,d
میشه 4*3*2*1 حالا جواب میشه تعداد همه حالت ها تقسیم بر تکراری ها یعنی:

[ برای مشاهده لینک ، لطفا با نام کاربری خود وارد شوید یا ثبت نام کنید ]

اوف ...! به داداشت بگو ترکیب بخونه خیال خودشو راحت کنه.!!:31::46:

zahedy2006
29-09-2008, 00:09
سلام اگر امکان دارد لطفاً در مورد فصل اول ریاضیات 2 توضیحاتی ارائه دهید به خصوص قسمت اول آن .
با تشکر


ریاضیات عمومی 2 دانشگاه منظورتون هست؟
بعضی کتب با دنباله شروع میشه بعضی با رویه های و نمودارهای سه بعدی. کدوم منظورتون هست؟!

shitan
29-09-2008, 09:02
سلام
من چند اثبات لازم دارم اگه ممکن است برام بنویسید
اگر a ,b شمارا باشد ثابت کنید A U B و A * B شماراست

مجموعه Z , Q شماراست

اگر F: از A به B و G: از B به C باشد
1) اگر F و G هر دو یک به یک باشد GOF نیز یک به یک است
2) اگر F و G هر دو پوشا باشد GOF نیز پوشا است

اگر F :از A به B تناظر یک به یک باشد معکوس F :از B به A موجود وخودش یک تناظر یک به یک است

atlanta
29-09-2008, 09:33
منظور من ریاضیات دوم دبیرستان بود

zahedy2006
29-09-2008, 10:25
تعیین علامت چندجمله ایها
تعیین علامت یعنی اینکه بفهمیم کجا علامت y مثبت و کجا منفی هست یعنی به ازای چه x هایی این علامت تغییر می کند.

در خطوط راست یعنی وقتی توان x در معادله 1 هست (با توجه به شکل کتاب) یک سمت ریشه دائما یک علامت و سمت دیگر علامت مخالف را دارد.
اگر ضریب x عددی با علامت مثبت مثلا یک باشد سمت راست ریشه (یعنی اعداد بزرگتر از ریشه) مقادیری اند که Y را مثبت می کنند یعنی به ازای آنها همواره y مثبت است و جهت دیگر منفی و بلعکس. یعنی اگر علامت ضریب x منفی باشد سمت چپ ریشه مثبت و سمت راست علامت Y منفی است.

برای راحتی می گویند: بزرگتر از ریشه موافق علامت ضریب x (که البته همون a است در y=ax+b) و کوچکتر از ریشه مخالف علامت a یا همون ضریب x

در عبارت درجه دوم:
عبارت درجه 2 به صورت سهمی هست.
اگه این سهمی کلا بالای محور X ها باشه معلومه که Y همواره مثبت هست
اگه این سهمی کلا زیر محور x ها باشه معلومه که Y دائما منفی هست.
یا کلی تر: اگر معادله ریشه نداشته باشد (دلتا منفی باشد) علامت Y موافق علامت a یا ضریب x^2 هست (در ax^2 + bx + c =y)
نکته: اگر دلتا مساوی صفر شود باز هم همین میشه به جز در نقطه ریشه مضاعف که اونجا y صفر هست و علامت نداره

اگر ریشه داشته باشد حتما دو ریشه دارد (دلتا بزرگتر از صفر)
مابین دو ریشه مخالف علامت a و اطراف موافق علامت a می شود

Maxwell_1989
29-09-2008, 16:12
من چند اثبات لازم دارم اگه ممکن است برام بنویسید
اگر a ,b شمارا باشد ثابت کنید A U B و A * B شماراست

مجموعه Z , Q شماراست

اگر F: از A به B و G: از B به C باشد
1) اگر F و G هر دو یک به یک باشد GOF نیز یک به یک است
2) اگر F و G هر دو پوشا باشد GOF نیز پوشا است

اگر F :از A به B تناظر یک به یک باشد معکوس F :از B به A موجود وخودش یک تناظر یک به یک است
همه رو برهان خلف بزن اگه نشد بیا پیش خودم!

shitan
29-09-2008, 20:15
همه رو برهان خلف بزن اگه نشد بیا پیش خودم!

راستش من زیاد سر در نمیارم اگر ممکنه خودت زحمتش را بکش:41:

Maxwell_1989
30-09-2008, 02:10
ببخشید شیتان جون!
یه چیزی گفتم بعد فهمیدم به این سادگی ها هم نیست.ان شاء الله ولی جواب میدم.به زودی...
یه پیام به میلاد بفرسی هم خوبه.خیلی حالیشه.
فعلا بای

m1367m2006
30-09-2008, 12:07
سلام برای اثبات شمارا بودن z برو اینجا

برای مشاهده محتوا ، لطفا وارد شوید یا ثبت نام کنید
برای Q اینجا نفهمیدی بگو توضیح میدم

برای مشاهده محتوا ، لطفا وارد شوید یا ثبت نام کنید
برای چنتا سوال اخر هم برو مجموعه بکش خیلی ساده حل مشن........
از ماکسول به خاطر تعریفش ممنون .
راستی تا یادم نرفته شما اصلا قضیه های همتوانی را میدونی

chessmathter
30-09-2008, 15:03
سلام برای اثبات شمارا بودن z برو اینجا

برای مشاهده محتوا ، لطفا وارد شوید یا ثبت نام کنیدبرای Q اینجا نفهمیدی بگو توضیح میدم

برای مشاهده محتوا ، لطفا وارد شوید یا ثبت نام کنیدبرای چنتا سوال اخر هم برو مجموعه بکش خیلی ساده حل مشن........
از ماکسول به خاطر تعریفش ممنون .
راستی تا یادم نرفته شما اصلا قضیه های همتوانی را میدونی
بهتر این توضیح ام بدیم:
مجموعه ای شمارا است که با مجموعه اعداد طبیعی یا زیر مجموعه ای از آن تناظر 1 به 1داشته باشد

m1367m2006
30-09-2008, 15:54
بهتر این توضیح ام بدیم:
مجموعه ای شمارا است که با مجموعه اعداد طبیعی یا زیر مجموعه ای از آن تناظر 1 به 1داشته باشد
فک می کنم کسی که این سوال را پرسیده میدونه چه مجموعه ای شماراست پس توضیح لارم نداره

chessmathter
30-09-2008, 16:17
فک می کنم کسی که این سوال را پرسیده میدونه چه مجموعه ای شماراست پس توضیح لارم نداره
حرف شما کاملا منطقیه.مارو دعوا نکن.!!!:31::20::10:

mahdi.a81
02-10-2008, 00:54
سلام دوستان
اين اولين مسئله اي هست كه من ارسال مي كنم اميد وارم خيلي زود بتونيد حلش كنيد ،

[ برای مشاهده لینک ، لطفا با نام کاربری خود وارد شوید یا ثبت نام کنید ]

سپاسگذارم

سلام
كافيه چندبار از اين معادله مشتق بگيري
اونوقت خيلي راحت حل ميشه
البته بايد به شرايط هم دقت كني
اميدوارم درست گفته باشم
من خودم حل نكردم
حل كن اگه اشتباه بود بگو

Parnyan
02-10-2008, 02:21
میشه یکی لطف کنه برد تابع رو برام توضیح بده ....اصلا بلد نیستم سوال حل کنم ...یه چیزایه کمی خوندم ...تو مدرسه هم هیچی بهمون نگفتن تو کتابام هم فقط سوال حل کرده توضیحه زیادی نداده :41::41::41:به شدت قاطی کردم ....:41:

اگه قانونی چیزی داره میگین ؟!!!



ALL THE BEST

Maxwell_1989
02-10-2008, 09:36
بله همین الان یه تاپیک باز می کنم برید ببینید.

Meisam Khan
04-10-2008, 15:35
سلام دوستان
بنده چند تا سوال داشتم.
البته تمامی سوالات زیر رو فقط با استفاده از تعریف حد اثبات کرد و از چیز هایی دیگه مثل دنباله و .... نمی تونم استفاده کنم.
لطفا هر چه سریع تر جواب رو بذارید.
با تشکر.
[ برای مشاهده لینک ، لطفا با نام کاربری خود وارد شوید یا ثبت نام کنید ]

Meisam Khan
04-10-2008, 15:53
سلام
زحمت این رو هم بکشید.
البته فقط با استفاده از تعریف حد(اونایی که به حد نیاز داره رو میگم.)
[ برای مشاهده لینک ، لطفا با نام کاربری خود وارد شوید یا ثبت نام کنید ]

M@STER
04-10-2008, 22:12
سلام دوستان چند تا سئوال داشتم اگه لطف کنید کمکم کنید کارم بدجوری گیره فردا باید جواب اینها رو ببرم..دستتون درد نکنه
این لینکه عکسه فقط 3 تا سئواله
[
برای مشاهده محتوا ، لطفا وارد شوید یا ثبت نام کنید
[ برای مشاهده لینک ، لطفا با نام کاربری خود وارد شوید یا ثبت نام کنید ]

chessmathter
04-10-2008, 23:35
سلام دوستان چند تا سئوال داشتم اگه لطف کنید کمکم کنید کارم بدجوری گیره فردا باید جواب اینها رو ببرم..دستتون درد نکنه
این لینکه عکسه فقط 3 تا سئواله
[
برای مشاهده محتوا ، لطفا وارد شوید یا ثبت نام کنید[ برای مشاهده لینک ، لطفا با نام کاربری خود وارد شوید یا ثبت نام کنید ]
ماشالا همه شبه امتحانیه.!!!:31::20::5:

[ برای مشاهده لینک ، لطفا با نام کاربری خود وارد شوید یا ثبت نام کنید ]

elendil
07-10-2008, 16:50
دوستان امکانش هست این سوال رو اثبات کنید

ثابت کنید حاصلضرب n عدد طبیعی متوالی بر n! بخشپذیر است

chessmathter
07-10-2008, 19:49
دوستان امکانش هست این سوال رو اثبات کنید

ثابت کنید حاصلضرب n عدد طبیعی متوالی بر n! بخشپذیر است


اول بگو قبول داری [ برای مشاهده لینک ، لطفا با نام کاربری خود وارد شوید یا ثبت نام کنید ] صحیح است یا نه؟اگه نه من اثبات کنم.
در این صورت n عدد متوالی رو

[ برای مشاهده لینک ، لطفا با نام کاربری خود وارد شوید یا ثبت نام کنید ]

در نظر بگیر در این صورت:

[ برای مشاهده لینک ، لطفا با نام کاربری خود وارد شوید یا ثبت نام کنید ]

و چون این عدد صحیح است پس عبارت اول هم صحیح است و حاصضرب n عدد متوالی بر !n بخشپذیر است.

elendil
11-10-2008, 19:53
حاجی جان من سوم هستم لطفا اگه میشه با توجه به مطالبی که تا الان خوندیم که فک کنم عاد کردنو اینجوز چیز میزا هستش اثباتش کنید این اثبات شما هم اون یه قسمت اولشو اگه میشه اثبات کنین و یه توضیح بدین :D:D:D

fereshteh-m
12-10-2008, 13:23
سلام آقای مدیر
استاد محترم ریاضیات
ازتون خواهشمندم در مورد تفاوت توابع ریاضی با نگاشتها که در درس نظریه ی گراف خوندیم برامون کمی بیشتر توضیح بدین.
خیلی ازتون ممنونم
در پناه حق تعالی

m1367m2006
12-10-2008, 13:40
ازتون خواهشمندم در مورد تفاوت توابع ریاضی با نگاشتها که در درس نظریه ی گراف خوندیم برامون کمی بیشتر توضیح بدین
اقا شرمنده این ماله دوره دبیرستانه یا درس توی دانشگاه؟
در ضمن فک میکنم اگه توی یه تاپیک جدا گونه مطرح می کردید بهتر بود

Maxwell_1989
12-10-2008, 13:53
با عرض سلام خدمت شما
البته من نه استاد ریاضیات هستم نه مدیر.
ولی فکر می کنم بتونم جوابتون رو بدم.
"توی بیشتر زمینه‌های ریاضی، اصطلاحات تبدیل و نگاشت با تابع هم معنی گرفته می‌شن.ولی توی بعضی از زمینه‌ها تبدیل و نگاشت ویژگی‌های دیگه ای علاوه بر تابع بودن هم دارن.مثلا توی هندسه، یه نگاشت(گاهی) بعنوان یه تابع پیوسته تعریف می‌شه."
با تشکر

mahsa1469
12-10-2008, 15:16
سلام
می شه عبارت زیر رو برام تعیین علامت کنید
cos(sinx
البته می دونم که در بازه ی 0 تا 2 پی جواب حقیقی نداره می خوام علامت این عبارت رو تو ناحیه های مختلف بدونم

mohammad96
12-10-2008, 23:40
سلام
می شه عبارت زیر رو برام تعیین علامت کنید
cos(sinx
البته می دونم که در بازه ی 0 تا 2 پی جواب حقیقی نداره می خوام علامت این عبارت رو تو ناحیه های مختلف بدونم

سلام،

برای مشاهده محتوا ، لطفا وارد شوید یا ثبت نام کنید به ازای تمام مقادیر x مثبته. چون Sin x در بازه منفی یک تا مثبت یک تغییر میکنه و Cos در این بازه همواره مثبته و مقادیر بین 0.5403 و 1 رو اختیار میکنه.

[ برای مشاهده لینک ، لطفا با نام کاربری خود وارد شوید یا ثبت نام کنید ]

در مورد
برای مشاهده محتوا ، لطفا وارد شوید یا ثبت نام کنید هم تغییر علامت مانند Cos هست. در مورد اخیر تابع بین بازه 0.8415- و 0.8415 تغییر میکنه.

chessmathter
15-10-2008, 21:18
دوستان امکانش هست این سوال رو اثبات کنید

ثابت کنید حاصلضرب n عدد طبیعی متوالی بر n! بخشپذیر است




اول بگو قبول داری [ برای مشاهده لینک ، لطفا با نام کاربری خود وارد شوید یا ثبت نام کنید ] صحیح است یا نه؟اگه نه من اثبات کنم.
در این صورت n عدد متوالی رو

[ برای مشاهده لینک ، لطفا با نام کاربری خود وارد شوید یا ثبت نام کنید ]

در نظر بگیر در این صورت:

[ برای مشاهده لینک ، لطفا با نام کاربری خود وارد شوید یا ثبت نام کنید ]

و چون این عدد صحیح است پس عبارت اول هم صحیح است و حاصضرب n عدد متوالی بر !n بخشپذیر است.







حاجی جان من سوم هستم لطفا اگه میشه با توجه به مطالبی که تا الان خوندیم که فک کنم عاد کردنو اینجوز چیز میزا هستش اثباتش کنید این اثبات شما هم اون یه قسمت اولشو اگه میشه اثبات کنین و یه توضیح بدین :D:D:D
اگه سومی (دبیرستان).!:31:
پس جز صحیح و ترکیب خوندی و میدونی این[ برای مشاهده لینک ، لطفا با نام کاربری خود وارد شوید یا ثبت نام کنید ] برابر تعداد زیر مجموعه های r عضوی از یک مجموعه n عضوی است.
برای اینکه نشون بدیم [ برای مشاهده لینک ، لطفا با نام کاربری خود وارد شوید یا ثبت نام کنید ]صحی ح است کافی است نشون بدیم که بزرگترین توان عدد اولpکه صورت را میشمارد از بزرگترین توان p که مخرج را میشمارد بزرگتر مساوی است.
فرض کن p عدد اول و n عدد طبیعی باشد آنگاه بزرگترین توان p که !n را میشمارد برابر است با:

[ برای مشاهده لینک ، لطفا با نام کاربری خود وارد شوید یا ثبت نام کنید ]

حالا طبق چیزهایی که بالا گفتیم باید ثابت کنیم :

[ برای مشاهده لینک ، لطفا با نام کاربری خود وارد شوید یا ثبت نام کنید ]

حالا طبق این ویژگی معروف جز صحیح :

[ برای مشاهده لینک ، لطفا با نام کاربری خود وارد شوید یا ثبت نام کنید ]

به ازای هر i داریم:

[ برای مشاهده لینک ، لطفا با نام کاربری خود وارد شوید یا ثبت نام کنید ]

پس:

[ برای مشاهده لینک ، لطفا با نام کاربری خود وارد شوید یا ثبت نام کنید ]

و حکم ثابت شد و [ برای مشاهده لینک ، لطفا با نام کاربری خود وارد شوید یا ثبت نام کنید ] صحیح است.

elendil
16-10-2008, 18:10
واقعا مرسی چس مستر جان :D:D:D:D:D:D:D

فقط یه بار هم اگه میشه اینا رو بدون سیگما ها بنویس :D:D:D:D

دستت درد نکنه عالی بود :D:D:D:D

پ.ن :

راستی شمردن رو هم نخوندیم یا اگه سال های قبل خوندیم یادم نمونده چیزی ازش :D:D:D:D

m1367m2006
16-10-2008, 18:47
میشمارد
برادر ضایع بازی درنیار
اینا اول دوم دبستان یاد میدن
یعنی عاد میکند .
بخش پذیر است
موفق باشی:40:

saeed_cpu_full
17-10-2008, 11:36
از اساتید میخوام که جواب این سؤال ها رو بدن

1- در یک صفحه ی شطرنج ( 8×8 ) چند مربع میتوان نشان داد؟
2- با رئوس یک 12 ضلعی چند مثلث مختلف می توان ساخت؟

chessmathter
17-10-2008, 12:31
از اساتید میخوام که جواب این سؤال ها رو بدن

1- در یک صفحه ی شطرنج ( 8×8 ) چند مربع میتوان نشان داد؟
2- با رئوس یک 12 ضلعی چند مثلث مختلف می توان ساخت؟

1)
[ برای مشاهده لینک ، لطفا با نام کاربری خود وارد شوید یا ثبت نام کنید ]

2) اگه منظورت اینه که مثلث ها ی مشابه رو حساب نکنیم فک کنم با یه شمارش ساده بشه 12 تا ولی اگه مشابه ها هم حساب میشه[ برای مشاهده لینک ، لطفا با نام کاربری خود وارد شوید یا ثبت نام کنید ]

elendil
18-10-2008, 14:14
برادر ضایع بازی درنیار
اینا اول دوم دبستان یاد میدن
یعنی عاد میکند .
بخش پذیر است
موفق باشی

منم فک میکردم عاد کردن بشه ولی نه دیگه با این کلمات :D:D:D:D

mahsa1469
18-10-2008, 16:04
سلام بچه ها من شکل تابع cosx بعلاوه ی قدر sinx رو می خوام رسمش کردم می خوام ببینم درسته یا نه خیلی برام مهمه

chessmathter
18-10-2008, 17:27
واقعا مرسی چس مستر جان :D:D:D:D:D:D:D

فقط یه بار هم اگه میشه اینا رو بدون سیگما ها بنویس :D:D:D:D

دستت درد نکنه عالی بود :D:D:D:D

پ.ن :

راستی شمردن رو هم نخوندیم یا اگه سال های قبل خوندیم یادم نمونده چیزی ازش :D:D:D:D

آقا عذر. گرفتارم .
ابن یکی از سیگما ها بغیش هم مثل همین

[ برای مشاهده لینک ، لطفا با نام کاربری خود وارد شوید یا ثبت نام کنید ]


یه مثال عددی بزنی حله.

mohammad96
19-10-2008, 00:17
سلام بچه ها من شکل تابع cosx بعلاوه ی قدر sinx رو می خوام رسمش کردم می خوام ببینم درسته یا نه خیلی برام مهمه

سلام،

[ برای مشاهده لینک ، لطفا با نام کاربری خود وارد شوید یا ثبت نام کنید ]

elendil
19-10-2008, 16:40
آقا عذر. گرفتارم .
ابن یکی از سیگما ها بغیش هم مثل همین


یه مثال عددی بزنی حله.

مرسی حاجی جان حل شدش همه چی :D:D:D:D

noia
22-10-2008, 22:21
سلام،
دو تا سئوال از گسسته (گراف) دارم:

1- مجموع مرتبه و اندازه یک گراف کامل 21 می‌باشد. تعداد مسیر به طول 3 در این گراف را حساب کنید.
2- چند گراف ساده با رئوس {a, b, c, d, e, f} می‌توان داشت که q=5 و ae مجاور (حتما وجود داشته) باشد و cd وجود نداشته باشد.
ــــــــــــــــــــــ
ممنون.

chessmathter
23-10-2008, 00:12
سلام،
دو تا سئوال از گسسته (گراف) دارم:

1- مجموع مرتبه و اندازه یک گراف کامل 21 می‌باشد. تعداد مسیر به طول 3 در این گراف را حساب کنید.
2- چند گراف ساده با رئوس {a, b, c, d, e, f} می‌توان داشت که q=5 و ae مجاور (حتما وجود داشته) باشد و cd وجود نداشته باشد.
ــــــــــــــــــــــ
ممنون.
1.اول p رو پیدا میکنیم

[ برای مشاهده لینک ، لطفا با نام کاربری خود وارد شوید یا ثبت نام کنید ]

بعد چون هر مسیر به طول 3 به صورت a,b,c,d است
ابتدا 2 راس از 6 راس انتخاب میکنیم که میشه [ برای مشاهده لینک ، لطفا با نام کاربری خود وارد شوید یا ثبت نام کنید ] (چون ab با ba فرق نمیکنه)
بعد دو راس وسط را به 4 *3 حالت میشه نوشت پس جواب میشه
4*3*[ برای مشاهده لینک ، لطفا با نام کاربری خود وارد شوید یا ثبت نام کنید ]
2.مجوعه یالها{...,ac,ab} دارای (2و5)c عضو است یعنی 10 تا ما از این 10 تا 5 تا عضو میخوام که یکی ae باشه و cd نباشه پس جواب میشه(8,4)c

mohammad96
24-10-2008, 20:04
با سلام،
رفع ابهام اين حد چه جوريه:

lim (x*Ln x) x->0

ممنون!

saeed_cpu_full
27-10-2008, 18:56
سؤال:

مجموعه ی زیر را با نماد های ریاضی نشان دهید
{ 1 ، 11 ، 111 ، 1111 ، ... }

منظور از نماد های ریاضی یعنی مثلا اینطوری بنویسیم:
{ X | X € Z ، 1<x<6 }

m1367m2006
27-10-2008, 20:51
با سلام،
رفع ابهام اين حد چه جوريه:

lim (x*Ln x) x->0

ممنون!
البته این برای زمانی که به صفر مثبت میل کنه
[ برای مشاهده لینک ، لطفا با نام کاربری خود وارد شوید یا ثبت نام کنید ]
[ برای مشاهده لینک ، لطفا با نام کاربری خود وارد شوید یا ثبت نام کنید ]

سؤال:

مجموعه ی زیر را با نماد های ریاضی نشان دهید
{ 1 ، 11 ، 111 ، 1111 ، ... }

منظور از نماد های ریاضی یعنی مثلا اینطوری بنویسیم:
{ X | X € Z ، 1<x<6 }
میشه یه ذره توضیح بدید

saeed_cpu_full
27-10-2008, 20:59
میشه یه ذره توضیح بدید

مثلا میگن مجموعه ی اعداد صحیح بین 8 و -1 رو بنویسید با نماد های ریاضی بنویسید میشه اینطوری:
{ X | X € Z ، -1<X<8 }

اینطوری میخونیم:
X به طوری که X عضو مجموعه ی اعداد صحیح باشه و X بزرگتر از -1 و کوچیکتر از 8 هست
پس مجموعه ی ما میشه این:
{ 0 ، 1 ، 2 ، 3 ، 4 ، 5 ، 6 ، 7 }

حالا سؤال من اینه:

مجموعه ی زیر را با نماد های ریاضی نشان دهید
{ 1 ، 11 ، 111 ، 1111 ، ... }

m1367m2006
27-10-2008, 22:19
اینطوری میخونیم:
X به طوری که X عضو مجموعه ی اعداد صحیح باشه و X بزرگتر از -1 و کوچیکتر از 8 هست
پس مجموعه ی ما میشه این:
ممنون داداش
من خودم رشتم ریاضیه میدونم علامت چیه و چی میخونن
منظورم اینه که ...........